Speed as well as accuracy is important in this section. Work quickly, or you might not finish the paper. There are no penalties for incorrect responses, only marks for correct answers, so you should attempt all questions. Each question is worth one mark.

Calculators are NOT permitted.

Here you will find all TSA Conclusions questions that have been written 2008-2019.

Sometimes poverty is defined in relation to average income. For example, people may be said to be poor if their income is less than 60 per cent of the average income for the This is not a legitimate definition, because it is a measure of inequality rather than poverty. This is obvious if we consider the consequences of using the definition. It means that a huge pay rise for middle managers automatically throws some people into poverty, even though their income has not changed. On the other hand, the definition implies that in a society where almost everyone is starving, no one is poor.

1. Which of the following best expresses the main conclusion of the above argument?
  • 2
    0

    Explanation

    The correct answer is C.

    The argument focuses on discrediting the definition of poverty in relation to average income. The argument goes on to talk about how this definition is more a measure of inequality, backing up the authors point with evidence of how huge pay rises effect those whose income hasn’t changed, and causes them to be defined as impoverish despite not having their income changed. Looking at the answer option E is irrelevant, option B is not a valid conclusion, and option D is not a conclusion of the paragraph, more of a point that was made to help illustrate the conclusion. This leaves us with options A and C. While both options are valid conclusions, statement A acts to support option C when using the therefore test. Leading us to the correct answer of C.

    Post Comment

    Socialist politicians are often taunted by their opponents for leading lifestyles similar to those of their capitalist counterparts. The theme of the taunts runs like this; ‘You object on socialist principles to gross inequalities in the distribution of wealth; yet you enjoy a higher personal standard of living than the majority of the population. Therefore you are not really a socialist’. But the jibe can easily be answered, for there is no hypocrisy in arguing, even from a privileged position, for a fairer and more equal society.

    4. Which of the following is a conclusion which can be reliably drawn from the passage as a whole?
  • 0
    0

    Explanation

    The correct answer is B.

    This passage talks about the issues surrounding socialist politicians, and the common criticism, how can they be for socialism if they themselves are wealthy. Although the article speaks around this criticism, it ultimately is finalised by saying that although this argument can be made, it is something that can be answered. Therefore, you need to identify that this argument is for socialist politicians, and how they can be both socialist and wealthy. This makes answer options A and E incorrect conclusions. In addition to this, we can say that answer option D is also incorrect as it is an irrelevant point to the arguments raised in the passage. Moreover, answer C is not covered or mentioned in the text, making it an unreliable conclusion to be drawn from the passage. This means that answer option B is the only plausible conclusion that can be drawn from the passage.

    Post Comment

    There is much evidence to suggest that cannabis has therapeutic uses for people suffering from conditions such as multiple Given this, the prescription of cannabis should be made legal. If doctors were legally allowed to prescribe cannabis, multiple sclerosis sufferers and others might be spared much pain. Moreover, if cannabis could be legally prescribed, it would be possible to conduct large-scale surveys to establish whether cannabis really is of benefit in such cases.

    11. Which of the following best expresses the main conclusion of the above argument?
  • 1
    0

    Explanation

    The correct answer is D.

    The positive nature of this argument clearly shows that the author is pro-cannabis legalisation, and details multiple reasons as to why. This therefore automatically rules out negative conclusions that are opposing all the content of the argument, answer option C. In addition to this, answer option A is not covered, or alluded to in the text, and can therefore also be ruled out. While answer options B and E are points made in the text, neither can be the accurate conclusion as both should be followed by “therefore, *the conclusion*” Using the therefore technique for the three plausible options (B, D and E), you can accurately conclude the answer is D.

    Post Comment

    Random drug-testing of prisoners was introduced in 1995 in order to solve the many problems associated with prisoners taking drugs. Since cannabis can be detected in the body up to a month after having been smoked, prisoners are tempted to switch to heroin, which stays in the system for only 48 As a result, since drug-testing was introduced, cannabis use has declined by a fifth whereas heroin use has doubled. Heroin is not only a much more damaging drug than cannabis, but it is also extremely addictive. There is evidence that heroin addiction encourages prisoners to intimidate others in order to pay for the drug.

    15. Which one of the following is a conclusion which can be drawn from the above passage?
  • 0
    0

    Explanation

    The correct answer is D.

    The passages tone overall provides you with a negative view of random drug-testing of prisoners, despite it being introduced to solve the drug problems within prisons. The passage talks about how although cannabis use has reduced, it has lead to a much more damaging and dangerous drug, heroin, to be used more. Based on this information provided we can rule out answer option B, as it is incorrect based on evidence in the text. While answer options A and E are not detailed in the text enough to be able to draw these conclusions. This leaves you with options C and D. While option C and D are very similar, option C states the issue cannot be solved by random drug-testing, as opposed to option D that states it has not solved the problem. When looking at this level of detail it is important to think about what the text really is telling you. It hasn’t stated that it can not ever solve the drug-problem, but that it has not solved the problem. Therefore, we would can conclude that option D is the correct answer.

    Post Comment

    The quantity of food produced has always fluctuated according to environmental conditions: gluts and shortages have been with us for five hundred years or more. In the past, the free operation of market forces limited the adverse effects of these fluctuations, usually with far more success than any attempts at planned responses. But now we are changing the environment irreversibly. These changes will not only be permanent, but will have such devastating effects on world agriculture that market forces alone will be unable to prevent a global famine. So it is now vital to seek some sort of planned response.

    27. Which one of the following summarises the main conclusion of the argument above?
  • 0
    0

    Explanation

    The correct answer is C.

    The article talks around how although there has been fluctuations in quality of food produced due to environmental conditions, it is an issue that is becoming more prevalent, and with the irreversible environmental changes that are now occurring, this could lead to issues such as a global famine. The article even states that it is now vital to seek a planned response to this issue. Looking at the answer options, answers D and E are points made, and therefore not the main conclusion. While this question seems hard to find the accurate conclusion for as all answer options are relevant, it is good to think about using the therefore test. In this case, answer options A, B, D and E all act as points that support answer option C as the main conclusion.

    Post Comment

    Scientists are now developing genetically modified crop plants that produce their own pesticide. But pesticides are only effective if they are not overused; if they are applied continuously for long periods, then the pests that they are supposed to kill develop resistance to them. Pesticides only remain effective against pests, therefore, if periods are left during which they are not used. The genetically modified crop plants will produce pesticides continuously

    28. Which of the following is a conclusion that can reliably be drawn from the above passage?
  • 0
    0

    Explanation

    The correct answer is A.

    The passage talks about the issues around effectiveness of pesticides and leads you to assume that these issues will be faced by genetically modified crop plants, as they continuously produce pesticides. Based on this information answer options B, C and D are ruled as incorrect. While answer option E is a conclusion that could be drawn from the text, as the crops producing their own pesticides would mean they are no longer required to be sprayed onto fields, however, as the passage is aimed at the ill effects of the use of pesticides, this is not an accurate overall conclusion to be drawn from this argument. Therefore, answer option A must be correct, as it is the only answer that takes all of the information provided in the passage into consideration in its conclusion.

    Post Comment

    All’s fair in love and war – and motor racing. That’s how some commentators view the failed attempt by a great driver to disable his main rival’s car by steering into it. Had he succeeded, he would have won the championship, because he was ahead on Fair? Why not? It may have been dangerous and irresponsible but it was not unfair. This driver had earned his one-point lead in the championship and was trying, like any serious competitor, to defend it. In a reversed situation, the other driver could – and arguably would – have done the same. Motor racing is a tough and uncompromising sport with huge incentives to win at all costs and all the drivers know what to expect.

    35. Which of the following best summarises the main conclusion of the argument above?
  • 1
    1

    Explanation

    The correct answer is A.

    The passage speaks about the competitive nature of motor raining, and the dangerous lengths that some drivers go to in order to score more points. The argument goes onto add that most drivers would act in the same way in they were in the same situation. However, provides mix feelings as to the fairness of such actions. Looking at the answer options, B and E, are all points that are raised within the text, and are likely to be followed by a therefore, leading to the conclusion, therefore showing that they are not the main conclusion of this passage. Option C is not a valid conclusion, as it disagrees with the passage. While option D does follow the paragraph, it is not covering all the points raised in the passage. The only answer option to addresses all points raised, and while may be not your opinion, does reflect the opinion of the paragraph.

    Post Comment

    Over the past few years in parts of the South West, a number of farm animals have been found killed in the fields. The nature of the injuries indicates that a large predator must have been responsible. Local people talk of a number of ‘big cats’ such as pumas being responsible, claiming that such animals have indeed been seen. Official investigations into these claims have dismissed them, concluding that any sightings have been of domestic rather than of big cats. But these investigations have failed to deal with the point that, even if the sightings can be explained in terms of domestic cats, the injuries on the carcasses of the farm animals cannot Big cats such as pumas must be responsible for these attacks.

    45. Which of the following best expresses the main conclusion of the above argument?
  • 0
    0

    Explanation

    The correct answer is A.

    The article offers some thoughts on how a number of farm animals have sustained injuries in fields, and how these are indicative of large predators. The article talks on how sightings of big cats have been dismissed, yet there still remains a question as to how these farm animals sustained such injuries, with the final line of the passage stating that big cats must be responsible for these attacks. This final line does act as a suitable conclusion to this argument, so lets take a look at the answer options. Options B and C are incorrect and therefore cannot be the conclusion of this passage. Option D isn’t explored in the passage, while option E offers a vague answer, that again isn’t really covered in the passage. This leaves you with answer option A, which agrees with the passage, and the concluding sentence of the passage, that big cats, such as pumas must be responsible for these attacks.

    Post Comment

    In the twenty-first century, many people aged over 65 are perfectly capable of working, and also are likely to live longer than individuals did in previous times. If people work after age 65, there is more chance that they will be financially self-sufficient when they do retire. Compulsory retirement ages have long been banned in the USA, and we should get rid of them in the UK also. This would be good for individuals and for the country. We should ignore those employers who complain that they would be unable to get rid of older and inefficient employees. After all, those who are really incompetent to do a job can be dismissed on the grounds of merit, rather than of age.

    1. Which of the following expresses the main conclusion of the above argument?
  • 0
    0

    Explanation

    The correct answer is D.

    The passage explores arguments to abolish compulsory retirements ages, giving examples as to why it is beneficial to both the individual, and the country, as well as giving the example of the USA who banned this long ago. The argument also discredits employers arguments that older employees are inefficient, as if they are incompetent at their job, they can be dismissed on these grounds alone, not age. Looking at the answer options, while answer A is valid, it is not supported by the information in the passage. While options B, C, and E demonstrate points raised in the text, none of these are valid conclusions, as they are all supporting evidentiary factors to support answer option D, that the UK should abolish compulsory retirement ages.

    Post Comment

    The effect of shortening degree courses at universities from three years to two would be that students would have two-thirds as much time to think about their subject. The result would be graduates whose understanding of their subject was shallower and whose intellectual development was much less extended. Any measure that leads to a lowering of the quality of graduates should be resisted by universities.

    4. Which one of the following conclusions can reliably be drawn from the above passage?
  • 0
    2

    Explanation

    The correct answer is B.

    The passage is arguing that shortening degree courses from 3 years to 2 years is going to cause a lowering in the quality of graduates produced, for various reasons outlined in the text. Ultimately saying that this should be resisted by the universities. Looking at the answer option A, it is a point raised in the text. C, is an irrelevant point, not raised in the text. D, again, is not raised in the text, and would be more of an inference than a valid conclusion. While option E isn’t too much of a stretch to conclude, it is still not highlighting the main objective of the passage. Answer B does this, and can be concluded as a therefore point following all the points raised in the passage.

    Post Comment

    There is widespread and justified concern about the reliance on expert opinion in law court cases. Where experts disagree on an interpretation of the facts, there is always the possibility that the more charismatic and persuasive expert’s opinion will prevail for this reason alone. Their reason for holding the opinions they do could be more to do with their own personal prejudices than their professional expertise. However, if we want justice to be done, we should distinguish sharply between this and expert evidence which is not reliant on interpretation. It would be a tragedy if key advances in tackling crime such as matching DNA were regarded with suspicion just because such evidence came from an expert witness.

    11. Which of the following is a statement of the main conclusion in the above argument?
  • 0
    0

    Explanation

    The correct answer is C.

    The passage covers thoughts on the concerns raised on the reliance on expert opinions in law court cases. The passage explores justified reasons as to how personal prejudices can have influence, as well as how someone being called on as an expert may be more persuasive and charismatic, making their opinion more influential on the outcome of the trial. The passage talks about the need to distinguish effectively between expert evidence and expert thoughts and interpretations. Answer option C covers this conclusion perfectly. While answer option A, D, and E are all points raised in the passage, and option B is spoken about but is irrelevant as a conclusion to this passage.

    Post Comment

    A dishonest act, such as stealing money from a handbag or cheating in school, may arise for one of many different reasons. But the reasons may be consistent with the personality of the individual involved. One child may steal, for example, in order to show off; another, to get money to support a hobby (or to get a birthday present for a younger sister); another, to express aggression or hatred against the person from whom he or she One child may cheat in order to avoid punishment for academic failure; another, because of personal ambition; another to express contempt for the system.

    15. Which one of the following is a conclusion that can be drawn from the passage above?
  • 1
    1

    Explanation

    The correct answer is A.

    The passage explores the reasons behind why individuals may be dishonest in situations where they may steal or cheat. It simply states a range of reasons, from trivial, to good intentions, or hatred. Looking at the answer options, B is irrelevant to the information provided. Whereas answer options C, D and E all illustrate points raised in the passage as to why children steal. Although they are points, neither is an accurate conclusion that can be drawn. Therefore leaving you with the only possible answer option of A, which follows the themes outlined in the text.

    Post Comment

    Over-qualification is sometimes cited as a reason for rejecting an applicant for a job. But to qualify means to reach a certain standard regardless of the amount by which that standard is surpassed. If someone holding a first-class honours degree is rejected for a job on the grounds that academic excellence is less important than, say, a caring attitude, this is not a case of over-qualification in one direction but of under-qualification in For this reason, applicants should not be rejected because they are over-qualified; any explanation for failure should always be in terms of shortcomings.

    27. Which of the following best expresses the main conclusion of the argument above?
  • 0
    0

    Explanation

    The correct answer is E.

    The passage explores why over-qualification is often used as the reason for rejection. Then it argues that there are no circumstances in which someone is actually over-qualified. So this means it shouldn’t be used as a reason for rejection on a job application. A and E are both valid in terms of this. if A is true it supports the statement so is an intermediate conclusion. However E is the main conclusion. B and C are both reasons given but not conclusions. They both support A and E. D is required for E to be valid

    Post Comment

    The so-called ‘baby-boomer’ generation (people born in Britain in the late 1940’s /early 1950’s) will enjoy a good chance of reaching a ripe old-age. They benefited from a diet that was austere but healthy when they were children and this has influenced their adult In addition, people in this age group have given up unhealthy activities such as smoking cigarettes. In contrast, the generation represented by those currently in their 20’s and early 30’s are in danger of experiencing serious health problems which would decrease their life expectancy. This is the ‘Big Mac’ generation whose childhood diet was high in fat and sugar, which they continue to crave. They also show worrying tendencies to indulge in ‘binge drinking’ and other health-threatening activities.

    28. Which of the following is a conclusion that can be drawn from the above passage?
  • 1
    0

    Explanation

    The correct answer is D.

    The passage talks about the differences between the diet and how the baby-boomer generation have a good chance of reaching old age, due to a healthy diet. Whereas, the Big-Mac generation going into detail about how the lifestyle factors are likely to cause serious health problems as they engage in health-threatening activities. Looking at the answer options, B and C are points that have been made in the text. Options A and E are although plausible, not covering all points raised in the text and therefore cannot be the conclusion as they can be followed by the therefore test and answer D, which can be shown as the main conclusion from the text.

    Post Comment

    It is possible to think that someone is right to do whatever serves his own ends, including murder, theft and deceit, and to admire his ruthlessness. But one cannot think that this is not morally The phrase ‘morally wrong’ is not meaningless. There are some things, such as those mentioned above, which would have to be so described by anyone who was not unaware of the meaning of the expression. That some things are morally wrong can be shown to be true, every bit as decisively as, for instance, it can be shown that snow is white. It is not true that every moral question is ‘a matter of opinion’, still less of taste, personal preference, or choice.

    35. Which of the following best expresses the main conclusion of the above argument?
  • 0
    0

    Explanation

    The correct answer is B.

    In this passage the author discusses the meaning of actions defined as morally wrong. They further talk about how some but not all moral questions are matter of opinion. Looking now at the answer options, A, D and E are points around individual issues raised in the text. Option C however does offer a more rounded option, it still does not express as much of the overall theme of the argument as answer option B.

    Post Comment

    In attempting to reduce the high injury toll from road offences such as drink-driving, successive governments have tended to rely on the deterrent effect of increasingly severe penalties. However, severe penalties can act as a deterrent only if potential offenders believe that the chances of being detected are moderate or high. Recent surveys have shown that people perceive the chance of detection for the offence of drink-driving as being very small.

    39. Which one of the following can be concluded from the passage above?
  • 0
    0

    Explanation

    The correct answer is A.

    The passage talks about how severe penalties are given to high injury causing road offences. The text discusses how surveys have shown that although these act as deterrents, people perceive the chance of being caught to be small. Looking at the answers, B is irrelevant, C is incorrect, and options D and E have not been discussed in the text. This leaves the only plausible answer option as A.

    Post Comment

    In Britain, wetland areas have traditionally been used for cattle farming, which requires drainage of the land to allow cattle to graze in summer. However, maintaining the marshy conditions in wetlands is vital to the survival of the varieties of wildlife and plants which are exclusive to such areas. Because the interests of agriculture and of conservation are thus in conflict in wetland areas, water levels should be regulated by an independent body to enable a balance to be achieved.

    45. Which of the following best expresses the main conclusion of the argument above?
  • 0
    0

    Explanation

    The correct answer is A.

    The passage discusses the use of wetlands for cattle farming, and how there is a dual interest in the land for both agricultural and conservation reasons. The argument then goes on to say that because of this the land is regulated by an independent body to enable a balance. Looking at the answer options, B and D cover points raised in the argument. Options and E are irrelevant to the argument, leaving you with the only viable option of answer A.

    Post Comment

    In the case of plastic drink bottles, recycling is not the best answer to the problem of garbage disposal. Ninety five per cent of soft-drink bottles sold in the United States are made of a compound of polyethylene (PET) which poses recycling problems. They are potentially carcinogenic if recycled into new food or drink containers. Recycled PET tends to absorb other chemicals before re-melting and these chemicals could leach into food or drink that is stored in a recycled container. However, if disposed of in landfill sites, plastic is not harmful to the environment. It can be squashed flat, so that it occupies little space and it is inert. Unlike other waste, it does not leach into soil or water, nor does it produce a dangerous build-up of methane.

    1. Which one of the following best expresses the main conclusion of the argument above?
  • 0
    0

    Explanation

    The correct answer is E.

    The text speaks about the recycling risks of plastic drinking bottles as they are mostly made of PET, in the US. The passage goes on to talk about how if added to landfill sites there is little to the environment. It is important to note that it is not claimed in the arguement that PET could not be recycled safely, rather that it is potentially dangerous. Looking at the answer options now we can say that answer option A and B are not plausible answers. C is also incorrect, as the carcinogenic claims are only about the recycling process, not about general use of the plastic. Option D is not explored at all in the text, therefore meaning the only possible answer option is E.

    Post Comment

    In 1997 the UK Government banned the owning of handguns after a man went on the rampage, killing several people, with a collection of legally owned firearms. In the time since then the number of crimes involving firearms as a whole has increased. The number involving legally owned firearms has gone up by 0.1% as compared to a 5% increase overall.

    4. Which one of the following could be reliably concluded from the above information?
  • 0
    0

    Explanation

    The correct answer is D.

    The passage talks about how despite the banning of handguns in the UK in 1997, there has been an increase in the number of crimes involving handguns. This includes both overall firearm crimes and also those involving legally owned firearms. Based on this although answer A does agree with the text, the use of the word simply makes this a less reliable conclusion. Answer options B and C are incorrect, and answer E has not been explored in the text. Therefore, answer D is the only possible option that offers an overall conclusion for the text.

    Post Comment

    The Richter Scale is used to measure the strength of earthquakes. It has the property that an increase of one unit on the scale represents a tenfold increase in the strength of the quake. Three earthquakes have the following values on the Richter Scale:

    X : 2.0

    Y : 3.0

    Z : 4.0

    8. Which one of the following statements about their relative strengths is true?
  • 0
    0

    Explanation

    The correct answer is B.

    Post Comment

    Many people imagine that large country houses in the eighteenth century had smooth lawns. But in those days there was no way of killing wild plants and weeds and, with the lawnmower not invented, the only method of cutting a lawn was by teams of gardeners with hand scythes; today’s weed-free close cuts were unthinkable. Given this, lawns must have been like flowery meadows.

    11. Which one of the following best expresses the main conclusion of the argument above?
  • 0
    0

    Explanation

    The correct answer is C.

    The passage discusses how that the perception of 18th century properties with smooth lawns was impossible to actually achieve, as there was no way of killing wild plants and weeds, plus the lawnmower hadn’t been invented so lawns had to be cut by hand scythes. Answer A, although plausible from the text is not an accurate conclusion to the argument in the text. Answer B is true, however is a point in the text rather than the main conclusion. Answer D is an unknown as this was not addressed in the text, while answer E is irrelevant and unknown. Answer C provides an accurate overview of the argument in the text and is therefore the only possible main conclusion.

    Post Comment

    15. Which one of the following conclusions is best supported by the data given above?
  • 0
    0

    Explanation

    The correct answer is B.

    The data provided looks at the thoughts of young smokers and parents perceptions of attitudes of eachother around smoking. Answer A is unknown from the information provided, as this is not something asked. Answer C is incorrect. Answer D is not covered in the data. Answer E is subjective and therefore not something that can be concluded from the data provided. Answer B can be concluded as true, and is the only viable answer option.

    Post Comment

    Store loyalty cards are designed to encourage customers to use one supermarket only. But all the major supermarkets now have loyalty cards, so people are able to possess a loyalty card for all of their local supermarkets. So it is pointless for the supermarkets to continue using them and they might as well abandon the costly schemes, because they cannot be the primary reason that people choose to shop at a particular store.

    27. Which one of the following best expresses the main conclusion of the above argument?
  • 0
    0

    Explanation

    The correct answer is C.

    The text discusses how loyalty schemes at supermarkets were originally created to encourage customers to use just that one supermarket, but as all have these schemes and customers use them for multiple different supermarkets they have become obscelete and are only expensive for the company, while now no longer achieving their aim. Looking at the answers, A, B, D and E are just points made in the text not conclusions. Leaving the only viable option as C, which concludes accurately the overall message of the passage.

    Post Comment

    The demand for blood donors is increasing all over the world. In Western countries, in particular, demand has been rising so rapidly that shortages have begun to appear. In all such countries, demand is growing much faster than rates of growth in populations aged 18-65 from whom donors are drawn. And, despite a massive research effort to find alternatives, it remains true that in medicine there is no substitute for human blood.

    28. Which one of the following conclusions can be drawn from the passage?
  • 0
    1

    Explanation

    The correct answer is E.

    The passage speaks as to how there is an increased demand for blood donors worldwide, but especially in Western countries. The passage also talks about some reasons for this short fall, and how that in medical uses, there is no substitute for human blood. Looking at the answers, although answer A and C are viable they are not conclusions, but points to be raised in the text. Answer B is incorrect, while answer D is not spoken about in the text. This leaves answer E, which covers the main point that the text focuses around, and with use of the therefore test can be shown to be the only possible conclusion from the answer options.

    Post Comment

    Although the general public are demanding action to reduce the number of pedestrians killed in road accidents in residential areas, road bumps in these areas should be abolished. Road bumps were introduced to residential areas because they were cheap, quick to implement, and showed that Local Authorities were acting to reduce road deaths. They are admittedly effective at restricting the speed of traffic but they also hinder emergency services. The London Ambulance Service has stated that they cause delays in reaching critically ill people and this results in 500 deaths per year from heart failure alone in the London area. Electronic speed signs, and more effective signing of speed limits in residential areas, would be just as effective at reducing pedestrian deaths.

    35. Which one of the following best expresses the main conclusion of the above argument?
  • 0
    0

    Explanation

    The correct answer is D.

    The passage talks about the use of road bumps to reduce the number of patients being killed in road accidents in residential areas. However, the passage talks about the impact of this on the London Ambulance Service and how in turn, while saving pedestrians, road bumps cause delays for critically ill patients from getting to hospital in time, resulting in around 500 deaths per year from heat failure alone in London. The argument goes on to suggest that electronic speed signs would more effective. Looking at the answer options, A was a point raised in the text, B was not mentioned, C, although true from the text is not the conclusion, and E is also a point raised in the text. This leaves answer option D, which does reflect main conclusion of the argument.

    Post Comment

    There has been a recent trend to paint speed cameras bright yellow so that they can be seen easily and so act as a deterrent to speeding motorists. This is the exact opposite of what should happen. If cameras are obvious, drivers tend to drive very fast between them and brake hard when they see one ahead, which is a very dangerous practice. If cameras were virtually invisible, drivers would have to keep within the speed limits all of the time for fear of being caught unawares.

    39. Which one of the following can be drawn as a conclusion of the above passage?
  • 0
    0

    Explanation

    The correct answer is B.

    The passage argues that speed cameras should not be painted bright yellow for various reasons discussed. Arguing that reduced visibility of cameras will make drivers keep within the speed limits. Looking at the answer options, A, although true, is not a conclusion. Option C, isn’t mentioned in the text, option D does follow the conclusions, but is also a point raised in the text. This leaves options B and E, option E, requires you to assume that drives care if they are caught speeding, and therefore doesn’t necessarily follow the conclusion of the passage. Therefore, the viable conclusion is B.

    Post Comment

    There is a proposal to change our passports from the present, rather imposing, book-type documents to small plastic cards, a proposal which should be rejected. The cards are seen as having many advantages. For example, they will be easier to fit into people’s pockets, something which will become more important as other countries move towards compulsory identity cards. But this supposed advantage of smaller size is actually a disadvantage. It is the very fact that passports cannot be slipped into a holidaymaker’s pocket (and from there into the sand on the beach) that makes us take special care of them. It is, after all, a very important document.

    45. Which one of the following best expresses the main conclusion of the above argument?
  • 0
    0

    Explanation

    The correct answer is B.

    The argument talks about how a new plastic card style passport would have many advantages, however the passage also talks about the disadvantages of something so small and easy to loose. Looking at the answer options, A is incorrect as not truly a conclusion you could draw from the passage, answer C is untrue, D is untrue, E is something that although may be your opinion, isn’t discussed in the text, while answer B is correct. This is because the passage starts by saying this change to passports should be rejected, and goes on to discuss why, therefore being the accurate conclusion of this passage.

    Post Comment

    11% of nurses across the country would like to leave the profession. However, this figure rises to 39% amongst nurses who are working alternating shift systems, which mix day, evening and night shifts. It was found that many nurses preferred one type of shift most of the time, whether that involved working early, late or through the night. Employers should urgently review staffing arrangements to avoid the compulsory, alternating patterns. This would make nurses’ working hours more compatible with their social and family lives. A further benefit would be a reduction in the cost of training new nurses to replace those leaving.

    1. What is the best expression of the main conclusion of this argument?
  • 1
    1

    Explanation

    The correct answer is A.

    This argument addresses the issues around shift patterns worked by nurses. It talks about how many nurses prefer one type of shift, no matter what the time, therefore employees should be able to avoid compulsory alternating shift patters. As this would be better for their lives outside of work. Looking at the answer options C and E are stated points in the argument, D is not raised in the text, leaving options A and B. Although both are valid conclusions if we use the therefore test, it is clear that answer option A is the main valid conclusion for the argument.

    Post Comment

    It is clear that the world is getting warmer. The only thing that could slow down this process of global warming, and avert ecological disaster, would be an immediate switch to green technologies. If global warming continues at the present rate, some species will be unable to live in the warmer conditions. They will need to move to cooler places than they now occupy, for example closer to the poles, or higher up mountains. Not all of these species will be able to reach cooler climates, because of the long distances they would have to travel. Some of those that could move would end up in overcrowded habitats, which would make successful breeding impossible.

    4. Which one of the following can be drawn as a conclusion from the above passage?
  • 0
    0

    Explanation

    The correct answer is E.

    The passage speaks about the ecological disaster that will occur as the world gets hotter. The text speaks around the issues of the rate at which global warming is occurring, and speaks about the effects of this on different species that request cooler environments. Looking at the answer options, A is a point raised in the text, where as B is a point alluded to in the text but not explicitly said. C is not explicitly said that this is the case, as it is specifically refers to endangered species, which are not mentioned in the text. Option B, is untrue, as contradicts the initial argument in the text, that we could slow down the process of global warming with the immediate switch to greed technologies. This leaves us with answer option E, which is the only valid conclusion in the answer options.

    Post Comment

    The differences between human beings are sometimes more striking than the similarities. There is a minority of people who, driven by a competitive urge to be the first or the best, undertake ambitious expeditions involving severe physical deprivations, high risk of fatality, and extreme isolation. Compare these people to the majority who prefer an easy, risk free life of comfort and we can see that one human being can be as different from another in their behaviour as two different species of animal. Any attempt to form general rules about human behaviour needs to bear this variation in mind.

    11. Which one of the following best expresses the conclusion of the passage above?
  • 1
    0

    Explanation

    The correct answer is C.

    The text speaks about the differences between individuals who are competitive and risk taking in their behaviour, in comparison to those who prefer a risk free life of comfort. It speaks about how the various between mind is something that needs to be thought about when creating general rules about human behaviour. Looking at the answer options, A is something stated in the passage, while B is an irrelevant point not raised in the text. D is again a stated fact from the text, and E is something that is completely irrelevant to the passage. This leaves us with the only possible answer option, C.

    Post Comment

    Similar effects have similar causes. The problem is deciding when one set of effects sufficiently resembles another to infer a similar cause. You say that the universe resembles a man-made object like a house, so its cause must also be similar, an intelligent designer, resembling human beings. But is the universe so like a house? It seems to resemble an animal or vegetable just as closely, so why should its origin not lie in reproduction or some vegetative process? The universe contains many classes of effects and objects, all with their different causes. There seems to be no compelling reason for concluding that the universe as a whole resembles one of these classes of effects or objects more closely than another.

    15. Which one of the following is a conclusion that can reliably be drawn from the above reasoning?
  • 0
    0

    Explanation

    The correct answer is B.

    This passage talks about how the universe resembles man-made objects, and speaks around this abstract interpretation. The text goes on to speak about how it doesn’t resemble any particular class, effect, or object more than another. Looking at the answer options, both A and C are based on points raised in the text, and therefore not valid. While answers D and E are untrue based on the points raised in the text. Therefore, the only valid answer option is B.

    Post Comment

    Britain’s property market is in the midst of a full-scale slump with house prices falling at their fastest rate in 15 years, according to a new study. But why is there an assumption that continuing rises in house prices is a good thing? Why not see the fall in house prices as a welcome change? House prices as a percentage of income are grossly overpriced. Low interest rates and cheap mortgages have led to the property market, particularly in London, being overvalued. Surely this will make houses more affordable. First time buyers will be happy beneficiaries of a housing slump. Others finding a silver lining in the gloom are those in rural communities for whom a house had seemed simply out of reach.

    27. Which one of the following is the main conclusion of the above argument?
  • 0
    0

    Explanation

    The correct answer is E.

    This passage talks about the perception of the property market in Britain, and how house prices are falling at the highest rate in 15 years, yet we perceive this as a bad thing. It speaks about how the property has becoming overpriced and this drop in property prices and cheap mortgages rates will be beneficial to first time buyers. Looking at the answer options, answer A, C and D, although true, are all valid and all contribute to the overall view of the conclusion, yet, are not an overall main conclusion of the text. While option B is not a valid conclusions as is just a fact stated in the text. This leaves us with answer E, which concludes the main overall view of the text. In addition to this, answer options A, C and D, if true, would all contribute to concluding that answer option E is the main conclusion of the text.

    Post Comment

    The average life expectancy in working class areas of London was significantly higher than in similar areas in other major cities in the first half of the nineteenth century. In London during this time drinking tea had become a part of daily life amongst the poor. The reason for this was a combination of availability and cost. Boats bringing tea to the country docked in London and a certain amount was sold off directly to local markets. At this time in the rest of the country tea was still regarded as a luxury, consumed only by the better off.

    28. Which one of the following conclusions is best supported by the passage above?
  • 0
    0

    Explanation

    The correct answer is C.

    The author speaks about different working class areas and how those in London drank tea during the first half of the nineteenth century, while elsewhere in the country tea was reserved as a luxury enjoyed by those who were wealthier. The passage is relating the middle class in London drinking tea, to the increase in life expectancy, in comparison to the working class from other major cities. Looking at the answer options, both answers A and B are quite drastic conclusions, and are reliant on a large amount of assumption and a direct causative relation, rather than that of possible correlation. While looking at answers D and E, which are both irrelevant and not spoken about in the text at all. This leaves the only realistic conclusion, that is supported best by the text as option C. Which also fits to the possible correlation that this article is referring to. For clarity on these questions, think about the difference between causation and correlation.

    Post Comment

    England is set to become the most crowded country in Europe as its population grows by a third over the next 50 years, according to official projections. There are currently some 50 million people in England, but in 50 years this could hit 68 million – 1,349 for every square mile. Opposition political parties, who have obtained the data from the Office for National Statistics, demanded restrictions on migration levels. The opposition spokeswoman for immigration said “This demonstrates the real pressure public services are being put under as a result of the failed immigration policy of the present government, which the electorate are increasingly recognising.” Continued unrestricted population growth will lead to extremist political parties benefiting at the ballot box.

    35. Which one of the following best expresses the main conclusion of the above argument?
  • 0
    0

    Explanation

    The correct answer is A.

    The passage discusses how population growth in England, caused by unrestricted immigration, if continued will lead to extremist political parties gaining a higher vote. The argument outlines some reasons as to why, with facts and figures to support their argument in the text. Looking at the answers, E is completely irrelevant. While answers B and C are based on points and opinions raised in the argument, however they aren’t full conclusions. They do support answer option D, which although is a valid conclusion to the passage, can still be used to support the main conclusion, A. To see between the two which is the main conclusion and which is not use the therefore test. A therefore D doesn’t work, but D therefore A does, meaning that answer A must be the main conclusion.

    Post Comment

    Many people who live near to mobile phone masts are worried that radiation from these masts will damage their health. Mobile phones themselves have also been suspected of emitting dangerous levels of radiation, and a report recommended that children should be discouraged from making non-essential calls. Emission of radiation from the masts is much lower than that from a mobile phone. A few seconds making a phone call is comparable to 24 hours of exposure from a mast. Radiation from both these sources is below the international guidelines as to what is a dangerous level, but no one can yet be certain if these guidelines accurately reflect the level of risk.

    39. Which one of the following can be drawn as a conclusion from the above passage?
  • 0
    0

    Explanation

    The correct answer is E.

    The text discusses the worries of many who live near mobile phone masts with regards to the radiation and possible damage to health caused by this. The text speaks about how tests have show the emissions are well below international guidelines from what could be considered dangerous, yet no one can be certain if these guidelines accurately reflect the real risk that they pose on people living nearby. Looking at the answer options, A is incorrect, as although mobile risk is mentioned as suspected, it is not said to be fact, therefore can not be an accurate conclusion. Options B and D are also incorrect for similar reasons, as they conclude that those living close to the mast must be over estimating the risk, yet as we are unable to evaluate the risk accurately in the text we can not conclude this either. Looking at answer C, it is incorrect, as the passage states there is less radiation from mobile phones than masts. Therefore, answer option E must be correct, it takes into account the evaluation of the risk and public perception, therefore being an accurate conclusion of the passage.

    Post Comment

    Although study leave in the run up to exams is traditional, and benefits well-motivated, organised students, recent thinking suggests that it is not the best way to improve results. Some students are required to do housework or help in the family business rather than studying. Many others spend time playing computer games, watching daytime TV or hanging around with friends instead of revising. Schools which provide in-school study areas with teacher supervision and a programme of booster revision sessions are getting more A*s and fewer Ds. It is time to ditch study leave.

    45. Which one of the following best expresses the main conclusion of the argument?
  • 0
    0

    Explanation

    The correct answer is D.

    The passage talks about the benefits and disadvantages of study leave. It speaks about how hard working students are benefited by this, but this is not the cases for all students for a variety of reasons outlined. The argument rounds by saying that schools providing in-school studying areas with a revision programme produces better results, concluding that we should abandon study leave. Looking at the answer options, A, B, C and E are all points raised in the text and all help to support the valid conclusion, but are not the overall conclusion on their own. While answer option D reflects the overall tone of the argument and is the supported conclusion by all the other answer options, therefore it must be the conclusion.

    Post Comment

    If people go to a foreign country, they should try to learn at least some of the language of that country because, whilst it is difficult to pick up a good command of a foreign language in a short time, learning just a little of a foreign language helps you to find out more about the country itself, its people’s customs and mannerisms. As well as this, it means that you can function more easily by being able to ask for directions or just by being able to order what you want at a restaurant, which is much less embarrassing than pointing and arm-waving.

    1. Which one of the following is an expression of the main conclusion of the above argument?
  • 0
    0

    Explanation

    The correct answer is D.

    The passage talks about how when travelling to another country it is good to learn a small amount of the language, not just for general function such as asking for directions, but to learn about the culture, and also be less embarrassing than pointing and arm-waving. Looking at the answer options, A, B and C are points raised in the text, in addition to this, C is untrue and doesn’t follow the conclusion of the passage. Option E, isn’t discussed at all, leaving option D, which is a valid conclusion for the argument of the text.

    Post Comment

    One in ten adults in the UK has had a body piercing somewhere other than the ear lobe. 28% of these experienced complications and 1% were admitted to hospital, according to a survey of 10,000 Body piercing is three times more common in women than men. Navel piercings are most common amongst women, whereas men are more likely to have a nipple piercing. Swelling, infection and bleeding are common side effects and tongue piercings are the most risky. Serious complications resulting in hospital admission mostly occur with piercings performed by non specialists rather than those carried out in a tattoo parlour or specialist piercing shop. In other countries, people have been infected with hepatitis B and C and HIV.

    4. Which one of the following can reliably be concluded from the information above?
  • 1
    0

    Explanation

    The correct answer is D.

    The passage discusses body piercings, including facts on the percentage of UK adults with piercings and those who experience complications requiring hospital admission. The passage talks about common types of piercings, as well as where you can get piercings done. It also speaks about serious infections you can contract in other countries from piercings such as Hep B and C, and HIV. Looking at the answer options, A and B are not spoken about in the text. Option C, is irrelevant as the survey was of 10,000 adults, which can be deemed as a sizeable study. Option E isn’t mentioned in the text, leaving option D, which can be concluded accurately based on the facts mentioned in the text. Try the therefore test, at the end of the passage add therefore D, and you can see this is the only viable conclusion.

    Post Comment

    How long will it be before we realise that force feeding our children with junk TV is just as bad as allowing them to eat nothing but junk food? Children are impressionable, we know that, so why do parents allow them to gain their insights into life through a TV screen? I’ve heard so many ‘concerned’ mothers and fathers complaining about playground violence, petty crime and early sexual awareness in their children, yet they allow them to watch all sorts of rubbish on Rather than blame outside influences over which they have no control, they should recognise they have answers closer to home – they’re called common sense and a remote control.

    11. Which one of the following best expresses the main conclusion of the above argument?
  • 0
    0

    Explanation

    The correct answer is A.

    This passage talks about children watching junk TV, and speaks to the argument that we should not allow children to watch junk TV. It speaks about how many parents are concerned about violence, crime and early sexual awareness, yet says they are unconcerned about what they are watching on TV. Speaking quite abruptly about how parents should be using common sense and a remote control. Looking at the answer options, B, although true based on the passage isn’t the main conclusion. This is the same for answer C. Looking at answers D and E, these are not true based on the text, and therefore not viable answers. This leaves us with answer option A as the only viable conclusion. If you practice with the therefore test you will see it is the correct conclusion.

    Post Comment

    When serious accidents such as train and plane crashes occur there are demands to identify and punish those Companies found guilty of corporate killing (“corporate killing” means the company as a whole is held responsible for deaths) in these accidents could be given huge financial penalties. But the point of punishment should be to ensure that safety systems are excellent and safety procedures are carefully followed. Financial penalties for the company do not have a great impact on individuals within the company, hence they do not make them feel a sharp sense of personal responsibility for safety. If the law allowed for jail sentences for the directors of these companies, this would provide the incentive to ensure the best safety systems and procedures.

    15. Which one of the following can be drawn as a conclusion from the above passage?
  • 0
    0

    Explanation

    The correct answer is C.

    The passage discusses corporate responsibility for serious accidents such as train and plane crashes where companies can be found guilty of corporate killing. It speaks about how company financial punishments does little to impact individuals within the company with little sense of personal responsibility for safety. The argument speaks about how allowing jail sentences for directors of these companies would provide incentives to provide the best safety. Looking at the answer options, A is incorrect, as does not follow what the argument is calling for. B is not covered within the text and is very opinion based. Option D, although following the theme of the argument, is very opinion based and is also incorrect as it is for safety, not achieving justice. Option E, is also incorrect, as this not addressed in the text, and therefore unable to be implied. Therefore, this leaves option C as the correct answer.

    Post Comment

    It is time to bury the independence issue. Our Scottish Government’s prime role is to spend its £30 billion wisely, delivering effective and efficient provisions for education, health, social services and a sympathetic business environment – all in a sustainable physical environment and inside a legal framework that facilitates their development. Its job is also to seek harmony, joint policy, compromise and understanding with the UK Government in Westminster, which is certainly not what is happening at the moment. Calls for independence are seriously compromising these We need a referendum on the issue now before serious damage is done, and so that the Scottish administration can concentrate their talents on the job they are in place to do.

    27. Which one of the following best expresses the main conclusion of the above argument?
  • 0
    0

    Explanation

    The correct answer is B.

    This article addresses Scottish independence, speaking about what is meant to happen between the Scottish and UK Government, yet this is not happening. The argument calls for a referendum on the issue and speaks as to why it calls for such drastic action. Looking at the answer options, A isn’t addressed in the argument, C also hasn’t been stated as being adequate finance, so we are unable to conclude this. Answer D is not addressed, and E is assumed from the text, but is not a valid conclusion, as it has not addressed this point directly. Therefore we can conclude that B is the correct answer, as it follows on as a valid conclusion from the points raised in the argument.

    Post Comment

    Everyone is exposed to low background levels of asbestos and other mineral fibres in the environment, without any evident risk to health. There is absolutely no reason to be concerned about Exposure to asbestos fibres in homes and other buildings where asbestos is present and in good condition is not normally significantly different from the background exposure.

    28. Which one of the following is a conclusion which can be drawn from the above passage?
  • 0
    0

    Explanation

    The correct answer is D.

    This passage talks about how everyone is exposed to background levels of asbestos in the environment, without risk, and speaks about how exposure to asbestos in buildings where it is in good condition does not cause significantly higher exposure to that of background exposure. Looking at the answer options, A is untrue based on the passage, B is an assumption that we could make, but is quite a big jump from the information offered in the passage. Option C has not been fully explored in the text, and E, while it has been alluded to, is not a conclusion that can be drawn. This leaves D, which is a valid conclusion based on the information provided in the text.

    Post Comment

    Surveys of public opinion reveal that many people object to the idea of selling bodily organs for Some object on religious grounds claiming that the body is a sacred and holy thing which must not be violated. Others object on aesthetic grounds simply finding the idea of selling human livers for transplant somehow unpleasant or distasteful. But the sale of bodily organs can not be morally wrong. Everyone has the right to do what they want with their own private property and each person owns their own bodily organs.

    35. Which one of the following best expresses the main conclusion of the above argument?
  • 0
    0

    Explanation

    The correct answer is D.

    This passage discusses the trade of human organs, morally and speaks about individual autonomy over their own body and organs. Looking at the answer options, A is not something discussed in the text, B is a point made in the text, but not an overall conclusion. C and E, although true based on the text, are not the main conclusions. Leaving answer option D, which follows the overall theme of the text, and therefore is the best expression of the main conclusion.

    Post Comment

    Contrary to what many people claim, Facebook cannot improve our experience of It may make it easier to increase our number of contacts, but there is a limit to the number of people we can keep in mind and with whom we can have a meaningful relationship. The number is 150, and is set by the size of our brains. Moreover, the quality of relationships depends on the amount of time we invest in them. If we increase our number of friends, the time devoted to maintaining each relationship will decrease, and our emotional connection with each person will decline. Meaningful relationships are about being able to engage in activities with others face to face. Communication via the internet is no substitute.

    45. Which one of the following best expresses the main conclusion of the above argument?
  • 0
    1

    Explanation

    The correct answer is E.

    The article speaks around how you can only have a certain number of friends, based on how we build and maintain meaningful relationships. It speak about time needed to maintain, emotional connections, and how if we have too many friends this will decline with them. The argument finishs by talking about how the intervnet is not substitute for fact to face engagement and activities in maintaining and building meaningful relations. Looking at the answer options, A is a partial conclusion, but does not reflect the overall message of the text. B, C and D are points raised, not conclusions, but all support the overall conclusion of the text, which is answer E. Linking back to talking about Facebook, which was the opening and concluding aspects to this paragraph, with the points in the middle supporting this conclusion.

    Post Comment

    This week a controversial chef is urging the great British public to sample a fabulous new But instead of a buying bonanza the chef’s comments have provoked outrage because the meat in question is horse. Even the staunchest meat-eaters are up in arms. Says a rival chef: ‘I would never eat horse meat and would never serve it to my customers. It’s not part of our food culture. It’s unthinkable.’ But aren’t we being just the teeniest bit irrational? After all, in France and Belgium, two of our closest neighbours, there are whole shelves of horse meat alongside the beef and chicken. Here we eat lamb and pig, duck, even deer, ostrich and kangaroo. If we eat these, what is logically different about horse? Nothing.

    1. Which one of the following most closely expresses the main conclusion of the above argument?
  • 0
    0

    Explanation

    The correct answer is D.

    This passage speaks about public opinions on eating horse. The argument overall is that there is no difference between eating other meats to that of horse, however it is still considered controversial in public opinion. Looking at the answer options, A, B and C are all points that are raised in the argument, yet are not conclusions of the passage. E, while is something raised in the text, is only a partial conclusion fro the overall argument. This leaves answer D, which is the correct answer, and distinguishable from answer E by using the therefore test.

    Post Comment

    The International Code of Zoological Nomenclature states that the earliest recorded scientific name for an organism becomes the official name. This seems fair, but the system is flawed. Textbooks show the sequence of fossil ancestors of the modern horse as: Pliohippus, Merychippus, Mesohippus and the first horse ancestor The last of these looks out of place. Hyracotherium was once, more sensibly, called Eohippus; this descriptive name means “dawn horse”. The suffix –hippus (horse) is consistent with others in the list. Eohippus also sounds pleasant. The name was changed when it was discovered that a fossil fragment that had been named Hyracotherium was really from the same animal that had, until then, been called Eohippus.

    4. Which one of the following conclusions is best supported by the passage above?
  • 1
    1

    Explanation

    The correct answer is C.

    The text talks about the flawed system of scientific naming for organisms from The International Code of Zoological Nomenclature. The passage goes on to talk about how there was the changing of names from Hyracotherium to Eohippus. Looking at the answer options, A is incorrect as the passage revolves around the scientific naming decisions. B is also incorrect as it is not stated which is most appropriate. Option D is also not able to be accurately concluded as this is an opinion based conclusion, while E is again opinion based and therefore could not be accurately concluded. Leaving answer C as the correct, as it is offering the balanced conclusion that the passage talks through.

    Post Comment

    Films are expensive to produce and must take a significant amount of money at the box office to cover costs before any profit is made. Thus filmmakers are primarily interested in making a film that will appeal to a large number of people and so make a lot of profit. This often leads to the production of superficial, violent films that attract a large audience. Therefore, the Government should invest in the film If it did that, filmmakers would not need to be motivated entirely by the aim for profit.

    11. Which one of the following best expresses the main conclusion of the above argument?
  • 1
    1

    Explanation

    The correct answer is A.

    This passage talks through the issues seen in the film industry with regards to funds, with expensive to make films being required to make a large amount in the box office. The article talks about how the government should invest in the film industry, to motivate filmmakers to not be motivated by profit. Looking at the answers, B and C are points made in the text, D is irrelevant, while E, although true, is again a point raised in the text, and is not an overall conclusion. E, along with B and C all support option A as the overall main conclusion of the argument.

    Post Comment

    Those in favour of coursework contributing to A-Level grades say that this is much fairer than assessment by exam only, as it means that students who are willing to work hard but who perform poorly in exams will have a better chance of doing However, this arrangement currently allows far more opportunity for cheating, casting doubt on its fairness. For example, there is a growing market for customised essays, available (for a fee) via the Internet. At the moment the only deterrent is a teacher’s vigilance, but while teachers might identify work that seems atypical for individual students, they won’t necessarily detect when, for example, a student has had an unacceptable amount of help from family or friends.

    15. Which one of the following conclusions is best supported by the passage above?
  • 0
    0

    Explanation

    The correct answer is A.

    The passage speaks about coursework at the fairness of its impact on A-level grades. The passage argues that coursework provides fairness, yet it also speaks about cheating being more comment, and how currently the only deterrent is a teacher’s vigilance for cheating. Looking at the answers, B, although reflecting the main point of the argument, isn’t the overall conclusion that was demonstrated. Option C, D and E are not fully discussed in the argument, and are based on points in the text. Therefore it can be concluded that answer A is the correct conclusion that is supported best by the passage.

    Post Comment

    There is a naive view that the problem of rapid population growth can be effectively solved if enough money is invested in family planning services. But it is no accident that the world’s highest population growth rates are in poverty-stricken sub-Saharan Africa. Child mortality in this region remains stubbornly high, so it makes sense for women to have lots of children to ensure that at least one son survives into their old age. Children are also economic necessities for peasant families engaged in traditional agriculture; in many systems the larger the family, the more land is allocated to it. It follows that only by reducing the demand for large families through economic development can the problem of overpopulation be solved.

    27. Which one of the following best expresses the main conclusion of the above argument?
  • 0
    0

    Explanation

    The correct answer is D.

    This passage talks around the misconception that problems caused by rapid population growth can be solved with investment in family planning. The argument talks about how child mortality in poverty-stricken areas of sub-Saharan Africa are still high and there is also the exonomic necessity for larger families to help manage agriculture. It also speaks about how the reducing demand for larger families can only be reduced by economic development, then the overpopulation will be solved. Looking at the answer options, A is not addressed in the text, B is again not fully addressed in the text, C is a plausible conclusion from a point raised, however it is based on assumptions. While answer E is a conclusion, it is only partial as it helps to support the main conclusion, which is answer D.

    Post Comment

    ‘Organic food is no healthier than other produce, scientists claim.’ This was the headline in an article published in 2009 in The Independent. It reported that a comprehensive review of 50 years of evidence showed no appreciable difference in nutrient levels between organic and conventional farm If the review is correct shoppers will think twice about buying organic food, especially if the price difference remains significant. Only if people really do receive some health benefit in return for the extra money will the current upward trend in organic sales continue.

    33.Which one of the following presents the strongest challenge to the above argument?
  • 1
    0

    Explanation

    The correct answer is C.

    This passage speaks about perceptions of organic food being healthier than conventional farm produce. It speaks about nutrient levels being no difference, yet customers are paying more. It speaks about how customers should only pay more if they are receiving additional health benefits, yet the upward trend in organic sales continues. Looking at the answer options, A is unknown, B is also unknown from the passage. D is again not fully addressed in the passage, and could be said is untrue based on the text, while E covers points and is a valid conclusion, however not the main conclusion, as it supports answer C, which is the main conclusion of the passage.

    Post Comment
    anon Medicmind Tutor

    Sat, 30 Sep 2023 20:52:29

    the question is which is the strongest challenge not which is the main conclusion?

    Sound economics may underlie the Government’s plans to put more prisons under private control, but the privatisation of prisons is nonetheless wrong. Although some countries observe basic human rights, they have the power to take away the liberty of individuals who break certain laws. This power is so far-reaching that only the State itself, acting through its officers, should be entrusted with it. Many prisoners have disturbed backgrounds and a contempt for authority, so exceptional disciplinary measures are needed to maintain order. The power to enforce such measures should not be left to private, profit-making organisations.

    35. Which of the following best expresses the main conclusion of the argument above?
  • 0
    0

    Explanation

    The correct answer is B.

    This passage speaks around privitisation of prisons. It speaks around how this is wrong and in some countries do observe basic human rights, yet they are able to take away the liberties of those imprisoned. It speaks about how the power to enforce measures on prisoners should not be done by profit-making organisations that are privately owned. Looking at the answers, A is only a point raised in the text, not a conclusion, C is an assumption based on some points, so not a valid conclusion, D again, is a point raised in the text yet not the main conclusion. E is also true, based on the text, yet is not the main conclusion, it supports answer option B, which reflects the overall nature of the argument.

    Post Comment

    The criminal law, armed only with punishment, is only ever capable of controlling a tiny minority of Communities can exert control over the behaviour of most people in much more positive ways: through the disapproval of one’s neighbours and fellow citizens and through a whole host of accepted social attitudes. Mass unemployment creates a large class of people who reject these attitudes and who are prepared to encourage, or at least tolerate, illegal ways of making a living, and hence has a profoundly subversive effect on society.

    45. Which one of the following best expresses the main conclusion of the argument?
  • 0
    0

    Explanation

    The correct answer is A.

    This passage speaks about how criminal behaviour is not tolerated by the masses, due to disapproval from fellow citizens, yet when there is mass unemployment there is a situation where the masses reject the previous attitudes and leads to encouragement or at least tolerance for criminal activity. Looking at the answer options, B is unknown, C is a point raised in the text, not the main conclusion, D is an assumption from the text, yet again not the main conclusion and E is true, yet again, is a point raised in the text that supports the main conclusion that is A. Using the therefore method, answer option A is the main conclusion that best expresses the argument.

    Post Comment

    In the past 15 years the government has poured millions of pounds into promoting sport to try to increase adult participation. The results show no lasting effect. A survey showed that unless children ‘catch the sport habit’ between the ages of seven and fourteen years they are unlikely to participate in sport as an adult, and that the sports they adopt as children are likely to be the sports they play as Schools need and should be given money to give children an interest in sport. Spending money trying to persuade adults to take up sport is like locking the stable door after the horse has bolted.

     

    1. Which one of the following best expresses the conclusion of the argument in the passage above?
  • 0
    1

    Explanation

    The correct answer is C.

    This passage talks about the efforts made to try increase adult participation in sport. The argument speaks of how schools need to be allocated funds to increase interest in sport, as spending money trying to persuade adults to take up sports will never work. Looking at the answer options, A, is not true, based on information in the text, B is potentially true, but we are unsure based on the text. D is potentially true, and alluded to, however we can not accurately conclude this from the passage. E is true but not always, leaving C as the only completely true conclusion based on the information provided in the text.

    Post Comment
    Fuck you Medicmind Tutor

    Sun, 30 Oct 2022 03:43:13

    B is eminently reasonable.

    The presence of an ethical symbol, such as ‘right’ or ‘wrong’, in a proposition adds nothing to its factual If I say to someone, ‘You acted wrongly in stealing that money’, I am not stating anything more than if I had simply said, ‘You stole that money’. In adding that this action is wrong I am not making any further statement about it. I am simply evincing moral disapproval. It is as if I had said, ‘You stole that money’ in a tone of horror, or written it with exclamation marks after it. The tone, or exclamation mark, adds nothing to the literal meaning of the sentence. It merely serves to show that the expression is attended by certain feelings in the speaker.

    11. Which one of the following best expresses the main conclusion of the above argument?
  • 0
    0

    Explanation

    The correct answer is A.

    Post Comment

    The funding of the BBC through a universal licensing system is considered by many to be outdated and There seem to be only a limited number of options for the continued funding of this public broadcasting network. Firstly, the licence fee system could continue as at present. Secondly, the BBC could raise its own revenue as a commercial company through advertising and sponsorship. Finally, the BBC could be paid for like other parts of the public sector, through general taxation.

    Funding of the BBC through taxation may raise questions about the BBC’s independence from government. This independence might also be put at risk if the BBC had to operate as a commercial company. It might be unduly influenced by its advertising and sponsorship customers unless serious safeguards are put in place.

    15. Which one of the following can be drawn as a conclusion from the above passage?
  • 0
    0

    Explanation

    The correct answer is C.

    The passage argues around the funding of the BBC, as it currently uses a universal licensing system. The argument talks of how this is outdated and unfair, and how it should use commercial revenues or used public sector funding from general taxation. However the argument talks about the independence of the BBC, and how this could be brought into question if there was funding through taxation or even as a commercial company. Looking at the answer options, A is untrue as outlined in the passage, B is true, but only if funding is changed from the current method of the universal licence. D is unknown and therefore not an accurate conclusion, while E is not fully explored in the passage and therefore could not accurately be concluded from the passage. This leaves C, as the only plausible conclusion that can be drawn from the passage.

    Post Comment

    We need to find solutions to the problem of high levels of carbon dioxide emissions. A number of solutions are being proposed but one of the most promising is that of using willow trees as a fuel for generating electricity. There are many environmental advantages in using willow. Burning wood has the environmental advantage that in doing so one releases into the atmosphere only as much carbon dioxide as the trees themselves absorbed. In addition, plantations of willows will not only support the insects on which songbirds feed, but also, being trees that thrive in wet areas, they will help to conserve our threatened wetlands. Other countries, such as Sweden, have already recognised the advantages of using willow for fuel. We should follow their example.

    27. Which one of the following best expresses the main conclusion of the argument above?
  • 0
    0

    Explanation

    The correct answer is B.

    This passage argues about solutions to help the high levels of carbon dioxide emission. One that is spoken about in detail is the willow tree and how it not only helps reduce carbon dioxide emissions, but helps to conserve wetlands too. Other countries such as Sweden already use willow for fuel as well, recognising its use. Looking at the answer options, A is a point raised in the text, C although true, is not the main point of the text, answer option D again is true, yet it is only a point raised in the text. Answer E is again true but acts to support the main conclusion that is B. If you use the therefore test, you can distinguish between answers E and B to show that answer B is the main conclusion of the text, and E supports this.

    Post Comment

    Brazilian police in Bahia recently ended their strike over low pay, which lasted 12 days and saw the region’s homicide rate double. During that time, politicians accused some police officers of causing panic by encouraging looting while on strike. Now the strike has ended they have offered an amnesty against the punishment of any striking officers as long as they did not commit any crimes during the For their part, the officers have received a 6.5 per cent pay rise and have complained that they were played off against the public and ended their strike so they could properly police the Salvador carnival. The Government contest this and say their decision to stop salary payments to any officers still on strike is what made the police return to work.

    28. Which one of the following can be drawn as a conclusion from the above passage?
  • 0
    3

    Explanation

    The correct answer is B.

    This passage speaks around how Brazilian police went on strike over their pay. This caused a great deal of crime, looting and homicide. It ultimately ended with a pay rise, but ended due to the Government wishing to have the Salvador carnival properly policed, things the government denies. Looking at the answer options, A is unknown but can be assumed, therefore is not an accurate conclusion. C is true, yet not the main conclusion of the passage. D is something that can be assumed, yet it again is not the main point of the passage. E is assumed but not full reflective of what the passage says, it is more points raised than overall conclusions. Therefore, the only viable conclusion for the passage is answer option A.

    Post Comment

    Acting on government advice, schools in England now provide only semi-skimmed milk to This assumes that all children are similarly at risk of becoming obese and ignores the benefits full-fat milk may bring to many children, including vegetarians and those who are underweight. It is another example of the Government interfering unhelpfully in the way people live their lives.

    35. Which one of the following best expresses the main conclusion of the above argument?
  • 0
    0

    Explanation

    The correct answer is C.

    This argument talks about schools providing pupils with only semi-skimmed milk in schools, due to the risk of obesity. It speaks about how the government has adviced this, ignoring the health benefits of full-fat milk to children. It also adds that it ignores those who are underweight, and states that it believes the government are being unhelpful and interfering in the way people live their lives. Looking at the answer options, A is not true, and is something explicitly stated as untrue in the argument. B is something that is true, but only a point raised in the argument. D is not something addressed fully, but can be assumed, therefore not the main conclusion, while E, is something that we can not be sure of. The passage is agreeing that milk is beneficial, yet doesn’t say that the effect of the governments advice will be negative on health of all. On some, yes, but not all, therefore not the main conclusion. This leaves answer option C as the only possible correct conclusion.

    Post Comment

    Many argue that successive government programmes to increase road safety have caused a progressive decline in the number of accidents on our roads. However, some observers have shown that the real number of accidents may be much higher than those officially recorded, with many accidents simply not being reported by They also point to the fact that, during the most recent period in which accident figures decreased, hospital admission rates for road accidents have remained fairly constant.

    39. Which one of the following can be drawn as a conclusion from the above passage?
  • 0
    0

    Explanation

    The correct answer is A.

    The passage states that the real number of accidents may be higher than officially recorded and that hospital admissions rates for road accidents have remained fairly constant. This suggests that the number of accidents may not have declined and that road safety may not have improved (as ‘many argue’ according to the first sentence of the passage). Therefore the argument is that this evidence does not support the idea that road safety has improved, highlighted by answer A.

    Post Comment

    Some people believe that raw food is healthier than cooked food because cooking destroys However, we should be grateful that our ancestors discovered how to cook. Researchers have found that people who stuck to a raw food diet for three years suffered from chronic energy deficiency and lost a lot of weight. Cooked food is easier to digest, so enables us to take in the necessary calories without too much expenditure of energy. Cooking also removes toxins from food, making cooked food safer. This has the result that our bodies are not very good at disposing of toxins naturally, but this is not a problem provided that our diet consists principally of cooked food.

    45. Which one of the following best expresses the main conclusion of the above argument?
  • 0
    0

    Explanation

    The correct answer is C.

    The second sentence states that we should be grateful that our ancestors discovered how to cook. Then the passage continues to give advantages of cooked food (safer, easier to digest, etc.). These, and the remainder of the passage, provide support for the statement given in the second sentence, which suggests that it is the main conclusion.

    Post Comment

    Heroin and cocaine are not the only illegal drugs with a terrible impact on global health. Counterfeit medicines can also do damage. Some of these fakes contain potentially toxic chemicals, and when taken to treat a disease they can kill. Others are like homeopathic remedies – while harmless in themselves, as they contain no active ingredients, they do not work as advertised. Efforts to tackle drug crime have prioritised illegal drugs and so have not dealt with crimes that are just as serious. This should now be remedied.

    1. Which one of the following best expresses the main conclusion of the above passage?
  • 0
    2

    Explanation

    The correct answer is E.

    Answer A is incorrect because cocaine and heroin are used as a comparison point to emphasise the damage counterfeit meds can do, which supports the main conclusion (E), eliminate A. Answer B is given by the first sentence of the passage, but gives support to the overall argument so is not the main conclusion. Answer C is indicated in the fourth sentence, but is not the main focus of the passage, so eliminate C. Answer D is given in the third sentence, and forms part of the argument by providing another reason for why E should be done.

    Post Comment

    Only John Lewis, the nationally adored and trusted UK retail group, can get away with the shamelessly sentimental marketing that they do. More than 100,000 people watched and rewatched on YouTube in the first week it was available their most popular ‘Always a Woman’ advertisement. If anyone else had dreamt it up, they would have been pilloried as sexist and outof-touch. In it, John Lewis offers a politically incorrect view of modern womanhood: pair-bonding and breeding are significant milestones, they say, not making CEO of a company. Surprisingly however, figures show that the retailer has magnificently outperformed every single rival.

    4. Which one of the following can be drawn as a conclusion from the above passage?
  • 0
    0

    Explanation

    The correct answer is E.

    Answer A is not stated not implied, so eliminate A. The writer does not suggest that the marketing was morally disgraceful, so eliminate B. Answer C is not mentioned in the passage. Answer D is an assumption, not the conclusion. Therefore E is the correct answer, because it gives the overall message of the passage.

    Post Comment

    Conspicuous consumption does not make us happy. Buying a flashy car sends the message: ‘I’ve got so many resources, I can afford to squander them.’ Paying more for organic foods signals another desirable trait: conscientiousness. We put too much effort into sending signals that don’t actually impress others and cause us stress. We evolved into emotionally wellbalanced individuals thousands of years ago. To be truly happy now, we should devote more time to the activities that dominated life back then: being at one with nature, being with children, or holding face-to-face conversations. We need to evaluate our lives and make sure they include many of those things we were doing 250,000 years ago.

    11. Which one of the following best expresses the main conclusion of the above argument?
  • 0
    0

    Explanation

    The correct answer is D.

    The first four sentences of the passage suggest answer A, but only forms part of the argument by suggesting that we can use our time better. The entire passage refers to the way we are acting, rather than our decisions, so eliminate B. The fourth and fifth sentences together indicate answer C, but the remainder of the passage discusses our actions rather than our emotions, hence C is only part of the argument and not the main conclusion. Answer E is not the main conclusion because you could say ‘E, therefore D’. Answer D is the main conclusion, and is emphasised in the final two sentences.

    Post Comment

    Currently 56% of British school children do not study a foreign language at all at GCSE, and seven university language departments have closed in the past seven years. People with only one language feel disempowered, whereas those who speak other languages are comfortable to travel the world. Once you can speak their languages, some of the mystique of foreigners goes away and you feel less intimidated by them. Good language skills are also important in the international business world.

    15. Which one of the following conclusions is best supported by the passage above?
  • 0
    0

    Explanation

    The correct answer is A.

    The passage starts by highlighting that the majority of British school children do not study a foreign language, then compares the advantages of being multi-lingual (being ‘comfortable to travel the world’) compared to the disadvantages of being uni-lingual (feeling ‘disempowered’). Hence A can be concluded from the text. Answer B cannot be inferred because the text mentions travel but not holidays specifically. Eliminate D because it opposes the argument given in the passage. The text does not mention the effect on language teachers, so eliminate C. Answer E cannot be concluded from the information given because we don’t know if there are anymore potential deals out there or if they could be being held back by other factors.

    Post Comment

    Increasing quantities of personal data are recorded and stored on computers: for example, where and when you use your mobile phone or what purchases you make in the supermarket. This poses a potential threat to our privacy. The Data Protection Act forbids the use of this information for a purpose other than that for which it was collected, apart from instances in which serious crime could be prevented or detected. We must not allow this principle to be abandoned because our liberty depends on it. Although at present there is no malign political purpose to which this information is put, we cannot be certain that this will never be the case.

    27. Which one of the following best expresses the main conclusion of the above argument?
  • 0
    0

    Explanation

    The correct answer is B.

    Answer A is indicated by the penultimate sentence, but is not the main conclusion because A can be use to support B as the main conclusion. C is unlikely to be the main conclusion because the passage does not mention the government. D is implied by the third sentence, but is only additional information that need not be provided to form the argument so cannot be the conclusion. The first two sentences of the passage are summarised in E, but like with answer A, E can be used to support B, so E is not the conclusion.

    Post Comment

    An intensely irritating fashion among politicians of all parties is to use the sound bite ‘doing the right thing’ to justify whatever action they have taken or propose to take. The phrase cleverly implies that if you do not agree with the action you are an immoral person – and no-one wants to be that. Taken to its logical conclusion, all we need are politicians who have some sixth sense that tells them what ‘the right thing’ is, and, hey presto, we can trust them to act on our behalf without any need for accountability. This is nothing short of dangerous brainwashing.

    28. Which one of the following can be drawn as a conclusion of the passage above?
  • 0
    0

    Explanation

    The correct answer is C.

    Answer A may be inferred from the first sentence, but the passage does not indicate that ‘sound bites’ fail to provide proper explanations, so eliminate A. Answer B is not a conclusion because the text says that politicians say they are doing the right thing, but this doesn’t mean that they think this themselves (ie they could be lying). The text overall points out the negative aspects of sound bites instead of politicians justifying their actions, suggesting that C could be a conclusion. The passage compares the effect of sound bites to brainwashing, which is different to ‘alienating’, so eliminate D. Answer E cannot be concluded because the passage says that politicians are using sound bites, but does not suggest this is their choice or preference.

    Post Comment

    Paying bonuses of at least £1 million to high-flying stockbrokers and employees of banks cannot be justified. Such massive bonuses encourage these high-flyers to take unacceptable risks with shareholders’ assets in the pursuit of profits. Furthermore, the system discourages our brightest young talent from taking jobs in industry, where the rewards are much more modest but where there is a great need for such talent. One of the reasons why City firms (i.e. London based financial organisations) can pay such bonuses is that they charge enormous fees for assisting with takeover bids between companies. City firms often encourage companies to engage in such takeover battles but the costs for these frequently unsuccessful battles fall on to the companies and thus on to the consumer.

    35. Which one of the following best expresses the main conclusion of the above argument?
  • 1
    0

    Explanation

    The correct answer is B.

    Post Comment

    Critics of homeopathic medicine argue that much of the apparent success of its remedies is due to the placebo effect (a placebo is a substance containing no medication). The remedies work because the patients believe they will work. In rebuttal, homeopaths often refer to a study involving a herd of pedigree Friesian cows. The herd was divided into two groups of 41 cows each. One group, the treatment group, was given a homeopathic remedy against mastitis, while the control group was given a placebo. The findings showed that 10 of the cows in the control group developed mastitis, while only one in the treatment group developed it.

    39. Which one of the following conclusions is best supported by the passage above?
  • 0
    0

    Explanation

    The correct answer is C.

    Answers A and B cannot be concluded because the passage only refers to a single study involving cows in which homeopathy was used, so a conclusion about homeopathy in general cannot be made. The passage only gives examples of homeopathy used on animals, but does not refer to potential use and benefit elsewhere so D cannot be concluded. The first sentence of the passage refers to critics of homeopathy, but does not imply that there are ‘a great deal’ of them, nor that their reasoning is ill-informed, therefore E is not a conclusion either. Hence C can be concluded.

    Post Comment

    Suppose a book begins with the sentence ‘Dark clouds gathered’. Does this mean that it is going to rain, or there is going to be a war, or something else no-one had thought of? The traditional answer was: ask the author, or try to work out what the author intended it to mean. But this would not give you the answer, surprising though that may seem. Why not? Because what the words mean is what they mean to you, the reader, and to anyone else who speaks the language to which they belong. Authors can’t change what words mean by intending something different, any more than you can. What they say they mean is no more correct than anyone else’s interpretation.

    45. Which one of the following best expresses the main conclusion of the above argument?
  • 0
    1

    Explanation

    The correct answer is C.

    Post Comment

    We need to accept flooding as a regular occurrence in the UK. Statistics show that days of ‘extreme rain’ (very heavy rainfall) have become more common since 1960, moving from one in a hundred days on average in the 1960s to one in seventy days in 2015. The apparent trend mirrors increases in extreme rain seen in other parts of the world. Countries such as India and China expect and prepare for this, but in the UK we have to change our thinking and plan our infrastructure to cope with it. The damage caused to homes and businesses as well as the risk of injury and death through flooding are very significant. Scientists say that, as the world has warmed by 0.7 °C, the atmosphere is able to hold 4% more moisture, which means more potential rain.

    1. Which one of the following best expresses the main conclusion of the argument in the above passage?
  • 0
    0

    Explanation

    The correct answer is D.

    The main conclusion is given by the fourth sentence of the passage. You may think answer A is implied in the fourth sentence, however A refers to an event in the past (‘The UK has been underprepared’) and the passage only alludes to future flood risk. Answer B is implied by the first clause in the fourth sentence, however the second clause of the same sentence suggest these countries are being used to emphasise answer D. The final sentence refers to global warming, but this is given as evidence to support that there will be regular flooding and does not mention politics, eliminating C. The final sentence tells us that the temperature has already risen, but gives no indication that this is to continue into the future, eliminating E.

    Post Comment

    There have been a growing number of calls to phase out the smallest denomination coins. The single penny is almost useless for purchasing items by itself. It would be simpler if prices were rounded to the nearest five pence and this would reduce the amount of small change carried around. But this view does not take into consideration that this will inevitably lead to price increases. Very few people would be concerned about paying £1 instead of 99 pence, but it is not the consumer who benefits from the 1% price increase. A single penny may not be worth much, but those who save them do eventually have enough to purchase something useful. There are many charities that also benefit from these small coins as donations.

    4. Which one of the following can be drawn as a conclusion from the above passage?
  • 0
    2

    Explanation

    The correct answer is E.

    The use of ‘also’ in the final sentence suggests that donation to charity is only part of the conclusion, eliminating A. The passage focuses on the benefits of small denomination coins, eliminating B and C which suggest the opposite view. Answer D may be tempting due to the penultimate sentence, but the final sentence suggests there are more benefits to small denomination coins than being able to save them. Therefore the answer is E, given by the two last sentences.

    Post Comment

    The apostrophe has limited usefulness as a punctuation mark, so it will be no loss if it disappears altogether from the English language. The fact that it can’t be heard at all in speech suggests we can make our meaning clear without it. It is already disappearing from much business correspondence without commerce grinding to a halt. The apostrophe is also widely misused, which is surely another measure of its unimportance. Many people apparently think it belongs in any word that ends with an ‘s’, as demonstrated by this sign seen in an off-licence window: ‘Beer’s, wine’s and spirit’s at pre-budget price’s’. Yet judging by the queue in the shop, nobody had failed to get the message that there were bargains to be had.

    11. Which one of the following best expresses the main conclusion of the above argument?
  • 0
    0

    Explanation

    The correct answer is B.

    Answer A is implied by the example given in the final two sentences of the passage, because the incorrect use of an apostrophe did not seem to affect communication. But this example is only given to support the main conclusion B, which is given in the second clause of the first sentence of the text. Answer C, like A, is suggested by the final two sentences and is again used to support B. Answer D is indicated in the second sentence, and also provides support to B. Answer E may be inferred from the passage, but it is not stated as clearly as B so is unlikely to be the main conclusion.

    Post Comment

    Scientists at University College London investigating the effects of bedtime routine on child development asked children to complete a number of tests including basic number skills, reading out word cards, and constructing designs from flat or solid shapes. Although the hour that children went to bed had little or no effect on their performance, having no set bedtime often led to lower scores. The effect was most striking in three-year-olds, where boys and girls scored lower on reading, maths and spatial skills tests than children of the same age who kept to a more rigid schedule.

    15. Which one of the following is a conclusion that can be drawn from the above passage?
  • 0
    0

    Explanation

    The correct answer is B.

    Answer A can be eliminated because it is too definite; the passage merely gives evidence to suggest, not confirm this theory. Answer C can be eliminated because the passage does not mention parents. Answer D can also be excluded because the text refers to bedtimes, not sleep quality. E is unlikely to be the answer because the passage does not mention cognitive disorders. This leaves answer B, which may be implied by the second sentence of the text.

    Post Comment

    There is no point to re-enactments of historical battles. Despite the meticulous attention to getting the uniforms right and the cannon smoke, the simulation of being there fails on every level: there is no real danger of death and you know what the outcome will be. It is merely an exercise in pageantry. In history-as-lived, the participants at Gettysburg did not know that the Confederates were not going to win, while in a scripted re-enactment everyone knows their role and destiny from the outset.

    27. Which one of the following best expresses the main conclusion of the above argument?
  • 0
    0

    Explanation

    The correct answer is C.

    You may initially think the answer is C because it is explicitly stated in the firsts sentence of the passage. However the text continues to explain why these re-enactments are pointless. This emphasised in the last sentence which compares a historic battle of un-anticipated events to a rehearsed re-enactment, suggesting this is the writer’s view and hence part of the argument’s main conclusion.

    Post Comment

    Crazy as it seems, it looks as if lead poisoning could be the major cause of the rise and fall of violent crime. Lead is so toxic that it is unsafe at any level and in Europe it has been banned from many products, including paint and petrol, since the 1990s. Lead poisoning in infancy, even at very low levels, impairs the development of those parts of the brain that regulate behaviour and mood. The effect is stronger in boys than in girls. Lead poisoning is associated with attention deficit disorder, impulsiveness, aggression and, some think, psychopathy. Studies between cities, states and nations show that the rise and fall in violent crime follows, with a roughly 20-year lag, the rise and fall in the exposure of infants to trace quantities of lead. Virtually all studies agree there is a strong correlation.

    28. Which one of the following could be drawn as a conclusion from the above passage?
  • 0
    4

    Explanation

    The correct answer is A.

    Post Comment

    The government’s decision to withdraw housing benefit from those living in council accommodation which has a spare bedroom has been a complete failure. It has failed to bring in sufficient savings to the government and has brought hardship to those affected by the cut. The government should focus less on punishing certain people to make savings and more on implementing affordable universal childcare, which would allow many more people into the workplace and able to pay taxes.

    35. Which one of the following best expresses the main conclusion of the above argument?
  • 0
    0

    Explanation

    The correct answer is D.

    The last sentence of the passage gives the overall conclusion, which is shown by D. The first two sentences help provide reasoning and support for the conclusion. A and B provide support to the conclusion. C can be inferred by the fact that the writer’s ultimatum is supposed to benefit ‘more’ people, and therefore cannot be the conclusion. E is incorrect because the final sentence indicates that the government should implement more childcare, but does not suggest that this will help save money.

    Post Comment

    Hospital food isn’t a joke. It’s a scandal. The list of meal options is ever longer, but there is nothing there that you would ever want to eat and nothing freshly cooked. These aren’t trivial issues. Because of this, over the past year people of all ages, but particularly the old, have faded away from lack of food. Despite being urged to eat, many simply won’t. Making sure patients who try to eat are actually nourished adequately is another problem. These days you need to be well-nourished before you enter hospital if you want to survive!

    39. Which one of the following is a conclusion that can be drawn from the above argument?
  • 0
    0

    Explanation

    The correct answer is D.

    The passage does not mention better systems so eliminate A. Nor does it mention nurses or supplements, so eliminate B and C respectively. The passage has acknowledged the lack of patient nourishment but does not indicate that this isn’t being well monitored already, so eliminate E. This leaves D, which is heavily implied by the second sentence, that the food nobody ‘would ever want to eat’ and there is ‘nothing freshly cooked’ which suggests the food is currently unappealing. The following sentences emphasise that this problem needs to be addressed.

    Post Comment

    The population pyramid below shows the age and gender distribution of the residents in England and Wales in 2010.

    England and Wales Population 2010

    44. Which one of the following statements is not true?
  • 0
    0

    Explanation

    The correct answer is D.

    Looking at the 9th bar from the bottom and all bars above it, there are more females than males so A is true. Looking at the 8th bar and all the bars below it, there are more males than females so B is true. Looking at the top bar, doubling the number of males gives 835800, which is less than the value given for females so C is true. The 10th bar from the bottom is the longest for both males and females, so E must be true. Therefore D must not be true, because the least numerous age group excluding <1 is over 85s.

    Post Comment

    Many people criticise what they call ‘the younger generation’ for what their seniors regard as immoral behaviour. These critics need to realise that because the world has changed, moral standards have changed with it. In fact, technology is advancing so rapidly that it is only the young who understand the world in which they live. It is they, not their parents or grandparents, who should be setting moral standards. Anyone over the age of 25 cannot help but be blinded by what they learned in the past. Their education has prevented them from perceiving and responding to the demands of the modern day, and so their moral judgments have no validity.

    45. Which one of the following best expresses the main conclusion of the above argument?
  • 0
    0

    Explanation

    The correct answer is C.

    The last two sentences state that the moral judgements of anyone over the age of 25 have no validity. The passage also says that ‘it is only the young who understand the world’. Therefore the conclusion is given by C. Answers A and E only form part of the argument leading to the conclusion. The first sentence of the passage implies the opposing view to answer D, so it cannot be the answer. B is stated in the second sentence and only forms part of the argument so is unlikely to be the conclusion.

    Post Comment

    Leaders of enemy countries are often described by our own political leaders and by the media as “mad”. But some of the features that we associate with madness can be useful assets in a political leader. Self-absorption, obsession, addiction to risk-taking and frenzied creativity all apply to some leaders agreed to have been “great”, as well as to those described as “mad”. People without some of those characteristics would be unlikely either to seek power or to be successful. The label “mad” is cynically used as a way of dehumanising and discrediting leaders of countries with whom we are in dispute.

    1. Which one of the following best expresses the main conclusion of the above passage?
  • 0
    0

    Explanation

    The correct answer is E.

    The text argues that traits associated with political leaders, such as addiction to risk taking or self-absorption can simultaneously be associated with ‘great’ leaders or ‘mad’ leaders and that as a result whether one uses ‘great’ or ‘mad’ to describe any one leader depends on their own political views. As a result options A,B,C or D are all incorrect and the correct option is E, as the last sentence summarises that the use of the word ‘mad’ can often be used to discredit other opposing leaders and that it is hence a propaganda tool as a opposed to a genuine description

    Post Comment

    In today’s 24/7 media environment, public figures have the ability to influence the behaviour of millions of young people, for better and for worse. There are celebrities who don’t give in to social pressures and are in the news because of good deeds which have a positive influence on young people, such as giving to charity. However, there are others who glamorise destructive behaviour such as drinking, smoking, swearing and sexual promiscuity, giving young people the impression that this is socially acceptable or even desirable. Although many celebrities maintain that they do not wish to be regarded as role models, it seems that this attitude is not only unrealistic but also irresponsible, as their behaviour will undoubtedly be imitated.

    4. Which one of the following can be drawn as a conclusion from the above passage?
  • 0
    0

    Explanation

    The correct answer is B.

    The text essentially argues that celebrities have the ability to influence the behaviours of others, since their activities are highly publicised in today’s ‘24/7 media environment’. It goes on to state that despite the fact that they’re seen as role models, some celebrities act quite irresponsibly. This leads us to the conclusion and the correct answer option of B, as the final sentence argues that regardless of whether or not they are happy to be seen as role models, their behaviour will continue to influence others and it is ‘irresponsible’ for them to assume otherwise. Answer A is incorrect, as is D as the text does not mention the role of parents. E is also incorrect as the final sentence does state that such a viewpoint is unrealistic as the behaviours of celebrities ‘will undoubtedly be imitated’. C is also incorrect as the text makes no mention of such a possibility.

    Post Comment

    Health clubs are booming. It is claimed that this must be a positive sign about modern society, in that people now appreciate that a healthy body is needed in order to sustain a healthy mind. But that claim is mistaken because what those optimistic conclusions fail to appreciate is people’s ability to deceive themselves. Surely if someone is really keen to stay fit they go jogging rather than join a health club? The health club phenomenon is just an example of how easily people can be swayed by marketing. They pay to jog on the spot when they could jog around the park for free. The vast majority of members barely use their health clubs and feel they are achieving something just by paying the fees.

    11. Which one of the following best expresses the main conclusion of the above argument?
  • 0
    0

    Explanation

    The correct answer is D.

    The answer to this question can be seen in the second/third sentence: “It is claimed that this must be a positive sign about modern society, in that people now appreciate that a healthy body is needed in order to sustain a healthy mind…But that claim is mistaken”. Option D summarises this claim. Regardless of what the writer writes about why people use health clubs, the fact remains that the conclusion of the argument is that memberships to health clubs cannot be used as a measurement of improvement in peoples’ health

    Post Comment

    Increasing attention is being paid to the environmental consequences of the plastic ‘single-use’ shopping bags provided to customers by supermarkets and other retailers. Not only does the production of these bags represent a wasteful use of natural resources, but there are also problems surrounding their disposal. For instance, the bags often end up as litter, and in the marine environment, they kill many animals that become entangled in them or mistake them for food. Numerous countries and individual cities have banned these bags, but experts on the environmental impacts of products warn that no perfect alternative exists. Whether using fabric or heavier plastic, manufacturing bags that are stronger and thus more reusable requires significantly greater quantities of natural resources than the vilified ‘single-use’ ones.

    15. Which one of the following is a conclusion that can be drawn from the above passage?
  • 0
    0

    Explanation

    The correct answer is E.

    The text refers to the negative environmental impact of producing and disposing plastic single use shopping bags and references an example of previous attempts to solve this issue by taking measures to ban the use of these products. The take-away part of the passage lies in the explanation for the effectiveness of this measure, showing that there is ‘no perfect alternative’ as the passage refers to the negative environmental impact of the alternative put into place. This shows us that there are pros and cons for either option and its necessary to consider all factors involved to determine a products environmental impact which lines up with option E. Option A, C and E are not mentioned throughout the passage so we can rule these out. Option B is incorrect as the text never mentions that the ban is ‘never’ an effective solution, be wary when an option includes ‘always’ or ‘never’ as these definite terms are usually won’t align with the passage.

    Post Comment

    The Human Rights Act (HRA) should apply to soldiers in the UK and at their overseas bases but not to those fighting in the heat of battle. The concern is that the HRA could damage the effectiveness of the armed forces. Commanders making split-second decisions in battle may be paralysed by fear that those decisions will become the subject of legal actions.

    27. Which one of the following best expresses the main conclusion of the above argument?
  • 0
    0

    Explanation

    The correct answer is A.

    It is important to note that the question is asking for the conclusion, not the argument. The conclusion is stated in the opening sentence: The Human Rights Act (HRA) should apply to soldiers in the UK and at their overseas bases but not to those fighting in the heat of battle. ” Thus, option A is the correct answer. Options B, D and E only apply to the reasoning, not the conclusion. Option C opposes the conclusion.

    Post Comment

    Increased wealth in many regions and an ever-growing population have produced an unprecedented demand for fish. This increased demand is occurring at a time when over 70% of the world’s marine fish stocks are now overfished or already fished at full capacity. One possible solution is the continued expansion of aquaculture, a long-standing means of farming fish for human consumption. Controls must be put into place, however, to manage the potential environmental risks associated with fish farming, such as water pollution and transmission of disease to adjacent wild fish populations. Moreover, the aquaculture sector has yet to devise effective methods for rearing some commercially popular predator species, such as tuna, without having to feed them wild-caught fish, which contributes to the pressure on our oceans.

    28. Which one of the following can be drawn as a conclusion from the above passage?
  • 0
    0

    Explanation

    The correct answer is D.

    “Aquaculture is a potential solution for increased fish demand if they can find a way to capture commercially popular fish” << main ideas summarised. A – The passage doesn’t talk about the comparison between fresh and marine fish. B – This is the opposite of what the question is trying to say. C – The extract doesn’t talk about alternatives to fish. D – Matches one of the main ideas (rear predator). E – This isn’t the main idea of the extract.

    Post Comment

    We need to change science lessons so that they give children a correct view of modern science. Children are often taught in science lessons that matter can exist in three different states – solid, liquid and gas. However, developments in science suggest that this is far too simple a picture. Increased knowledge of the universe has shown that many other states of matter exist, for example, plasma and liquid crystal. Plasma is thought to be the state of 99% of matter in the universe, so the traditional states of matter are in a very small minority. So the traditional view is giving children a false, indeed unscientific, view of the universe.

    35. Which one of the following best expresses the main conclusion of the above argument?
  • 0
    0

    Explanation

    The correct answer is E.

    The conclusion is the opening statement: we need to change science lessons so they give children a correct view of modern science. The argument underlying this demonstrates that this is because what is currently taught is incorrect. Thus E expresses the conclusion. With questions like this you have to be careful to identify what the conclusion of the paragraph is – it is not always the final sentence! Once you have identified this, working out the question becomes a lot simpler.

    Post Comment

    A pioneering scheme started two years ago in Cambridgeshire assigns an individual caseworker to the most vulnerable homeless people. The caseworker helps the homeless person navigate through the health, justice and housing systems until his or her life improves. It has already shown that an average of £1000 a month per homeless person has been saved – money that would otherwise have been spent on providing emergency support.

    39. Which one of the following is a conclusion that can be drawn from the above passage?
  • 0
    0

    Explanation

    The correct answer is A.

    The paragraph shows that assigning a caseworker to individual homeless people will prevent them from having to access emergency support and will improve their living situations. Thus it is appropriate to draw the conclusion that we should assign caseworkers to individual homeless people. Option D also appears correct, but this would be more appropriate as further evidence to support the use of individual caseworkers.

    Post Comment

    In the 1990s, there were two lines of argument against buying properties to let them out instead of living in them yourselves. One was the moral argument that you were driving the boom for your own profit and making houses unaffordable for everyone else; the other was the practical argument that houses were a risky investment. Despite these objections, the buy-to-let market continued to grow and now, twenty years later, the result is that the housing market is totally inaccessible to most of a generation. Rents in London have gone up eight times faster than wages. Plainly, no social good will come of this. Rents will continue to rise, and people who live in London will see their finances become more and more precarious.

    45. Which one of the following best expresses the main conclusion of the above argument?
  • 0
    0

    Explanation

    The correct answer is C.

    The passage focuses on the negative implications of buy-to-let on people. The phrase ‘no social good will come of this’ in the penultimate sentence is preceded by the word ‘Plainly’, which shows that the writer wants to highlight this as the main point of the argument.

    Post Comment

    Fear of death prompted us to develop technology that helps people live longer. But those same technological advances have condemned us to fear of infirmity and dementia, since so many of us will live to an advanced age. So as to eliminate this new fear, once we have reached old age we should be assisted to die if we choose death over a sad decline. According to current estimates, dementia affects almost 50 per cent of people by the age of 85, and bodily infirmity is guaranteed to develop further the older we get. There is no cure for old age.

    1. Which one of the following best expresses the main conclusion of the above argument?
  • 2
    0

    Explanation

    The correct answer is B.

    This answer needs you to identify that euthanasia is the same as assisted death. The article talks through reasons for euthanasia in older age, therefore leading us to the conclusion that B is the correct answer.

    Post Comment

    The world is now producing more farmed fish than beef. In 2012, fish production reached a record 66 million tonnes; cattle-farm output levelled out at 63 million tonnes. This ought to be good news for the environment. Cattle farming is one of the biggest contributors to global warming, while fish farming – as well as being sustainable – produces little pollution. Fish such as the silver carp can be reared on existing rice paddies, and they feed on grass, plankton and detritus. Species such as salmon, however, are carnivorous and are fed on smaller fish like anchovies caught in the wild. As a result, salmon can only be farmed by further depleting wild fish stocks.

    4. Which one of the following can most reliably be drawn as a conclusion from the above passage?
  • 1
    0

    Explanation

    The correct answer is D.

    The article is talking about how fish farming is better for the environment than cattle farming, as well as going on to talk about the difference between carnivorous and non-carnivorous. Combining the two these will give you the most accurate conclusion that can be drawn as answer option D. As although fish farming is helping the environment, it would be better for it to be non-carnivorous fish, not fish like salmon, that will further deplete the wild fish stocks.

    Post Comment
    nills Medicmind Tutor

    Wed, 05 Oct 2022 16:44:05

    why is not e

    Evidence suggests that children whose parents are married tend to fare better – financially, academically and socially – than children of unmarried parents. The government is therefore proposing a tax break for married couples, whereby married couples are reimbursed by approximately £150 per person per annum. This idea is flawed for several reasons. First, it punishes those whose partners have left them and broken up the marriage. Secondly, it punishes those who, through no fault of their own, have been unable to find a partner. Thirdly, it assumes that people in unhappy relationships will persevere in their unhappy state for something like £12.50 a month. This is patently absurd. Finally, it makes the bogus assumption that maintaining a poor marriage will be better for the children than separation.

    11. Which one of the following best expresses the main conclusion of the above argument?
  • 0
    0

    Explanation

    The correct answer is E.

    The article focuses around the idea, and even states that it is flawed for several reasons, going on to expand on this. This demonstrates how that the article is focus around this point. Making answer option E, the government’s idea of giving married couples tax breaks is flawed, the main expressed conclusion of the article.

    Post Comment

    The award of a Nobel Prize in economics, but not in psychology, sociology or anthropology, seems to imply that economics is not a social science, but an exact one, like chemistry or physics. Underpinning this idea is the thought that the human world operates like the physical world. Economists themselves perpetuate this impression by dealing in hard quantitative data, as opposed to observation and qualitative analysis. The defining feature of the natural sciences is their consistent success in making predictions, but almost all economists failed to predict the 2008 financial crisis. The striking collapse of LTCM, a hedge fund set up by two economists who were later awarded the Nobel Prize, further underscores the limits of economics to make accurate forecasts.

    15. Which one of the following can be drawn as a conclusion from the above passage?
  • 0
    1

    Explanation

    The correct answer is C.

    The article focuses around the debate on if economics is classed as a true science or a social science. Looking through the answer options, options A,B and E are all obviously incorrect based on the text. Option D although may be an accurate conclusion you can draw from the text, the main focus of the text is around the classification of economics as a science in relation to the Nobel Prize. Therefore, this answer option can not be true. This leaves you with option C which is the only answer option that follows a conclusion of the text.

    Post Comment

    Long-term unemployment is a big problem for the UK, both for the government and for the individual. The solution is unlikely to be one the government wishes to consider: invest money in improving the skills of job centre staff. Some people argue that providing welfare benefits makes unemployed people less motivated to find work. Other people, mostly outside the government, think that not enough is done to provide the help needed to get people into work. Neither side is right. Unemployed people are all different: some really want to work and would do so if they had help; others are content to live on benefits. We have to steer away from one size-fits-all solutions and tailor the approach to the individual. But this needs much higher levels of skilled staff than job centres currently have.

    27. Which one of the following best expresses the main conclusion of the above argument?
  • 0
    0

    Explanation

    The correct answer is D.

    The main conclusion is touched on at the beginning of the text, with the rest of the text acting to support this conclusion. This is that, the solution is to invest money in improving the skills of job centre staff. This is then further discussed in the text and explored as to why this does and doesn’t’ help. The text even goes as far to emphasise this point at the end of the paragraph, making certain that the correct answer option is D. The way to reduce unemployment is to spend on skilled staff in job centres.

    Post Comment

    A Prime Minister has a difficult judgement to make when deciding whether to sack one of their ministers or not. Faced, for example, with a minister involved in some kind of scandal, the Prime Minister might feel they should sack them to show they stand up for integrity and high moral standards, or, on the other hand, allow them to remain in post for other reasons such as loyalty to a colleague or a belief that they are, on balance, doing a good job. Sacking ministers shows that the Prime Minister is prepared to live by principles, even if by so doing they lose valued colleagues. The electorate will judge the Prime Minister to have acted with integrity: that is an invaluable perception for anyone wishing to remain in power.

    28. Which one of the following can be drawn as a conclusion from the above passage?
  • 0
    0

    Explanation

    The correct answer is A.

    Although the article talks around both sides of the argument, the overall opinion you gain from the article is that it is in the Prime Minister’s best interests to sack colleagues, even at the expense of loosing a great colleague. This is in the interest of the party and the perception given off in order to stay in power. This overarching conclusion follows answer option A. It is clear that answer options B, C, and D are incorrect, and option E does not convey the overall conclusion of the text, therefore A is the correct answer.

    Post Comment

    The current advice from the NHS in the UK is to eat five portions of fruit and vegetables a day. However, a recent study suggested that at least seven portions are needed to maximise health. Given that less than a third of adults in the UK, and even fewer children, currently meet the five-a-day target, there is little to be gained from changing the NHS advice to a higher number. A higher target might even be counter-productive, as evidence shows that a change in diet is best sustained when the change is made slowly and incrementally.

    33. Which one of the following best expresses the main conclusion of the above argument?
  • 0
    0

    Explanation

    The correct answer is C.

    The article talks around how new evidence has suggested to increase the current guidance on fruit and vegetable intake, however, it goes on to add that this could be counter-productive and shows the negatives of such an increase. Following this theme we can rule out answer options A,B and E, as they do not follow the conclusions we can draw from the text. Answer option D, although is plausible, it is implied and would not be the main conclusions the text is expressing. Therefore, the correct answer is C.

    Post Comment

    There seems to be universal agreement on Shakespeare’s greatness, yet many of his plays lack any significant development of plot or character. An exceptional skill with language is evident in all his works and his running theme concerning the relationship between appearances and reality has appealed to audiences across times and cultures. But is this enough to confer greatness? More likely this status derives from a small number of his plays, the four great tragedies, which succeed in dramatising reflection on the depths of the human condition. It is this which touches the hearts and minds of his audience.

    35. Which one of the following best expresses the main conclusion of the above argument?
  • 0
    3

    Explanation

    The correct answer is D.

    Hypocrisy “can be considered a virtue” and as a hypocrite you “still accept the moral principles you have failed to follow” hence the main matching conclusion is D. A – this is a point contributing to the main conclusion. B – This is only touched on once and again a point contributing to the main conclusion. C – this isnt even one of the points made in the passage. E – Its mentioned as a moral strength in the extract

    Post Comment
    Student Medicmind Tutor

    Sun, 30 Oct 2022 19:00:19

    a different question was answered here :/

    Modern aeroplanes are for the most part ‘flown’ by autoflight computers. These computers can take over when the plane is just 30 metres off the ground, maintain whatever speed and height is tapped into the flight management system, and land the aircraft automatically. But when autoflight computers experience situations they haven’t been programmed to handle – sudden structural damage to the aircraft, or extreme weather – they can throw back responsibility to the pilot, who may become confused over the level of control they have. The confusion is made worse by flight deck computers overloading pilots with a blizzard of alerts. This is when crashes are most likely to happen.

    39. Which one of the following is a conclusion that can be drawn from the above passage?
  • 0
    0

    Explanation

    The correct answer is A.

    The passage talks about how modern aeroplanes work and where errors occur and how this can cause to crashes, putting this down to both computer error and the computer overload to the pilot.

    Statement B is not correct. This is because the text does not allude to how well autoflight computers can be refined.

    Statement C is not correct. The text does not say it is unsafe in most situations, just that it is possible for humans to get confused by the outputs of computers.

    Statement D is not correct. The text does not discuss control of computers from the ground.

    Statement E is not correct. There is no discussion of entering data.

    Statement A is correct. While the text does not directly say this, the implication is that it is the confusion caused when an autopilot hands control to the pilot that causes problems. It is therefore a logical conclusion that if the system was developed to the extent that pilots were not required (again, the feasibility of this is not mentioned but the statement does not lean on feasibility) then it would be safer.

    Post Comment

    Charities address the negative effects of market economies, which apportion goods to people based on their ability to pay as opposed to their needs. The work of charities conceals the true impact of the market system and, worse, leaves the market in place to produce further negative outcomes. Charities therefore effectively perpetuate the problems the market system creates. Since it is immoral to support a practice that perpetuates the very problems it is supposed to resolve, giving to charity cannot be morally justified. After all, if the effects of a system are morally wrong, then it is immoral to sustain that system.

    45. Which one of the following best expresses the main conclusion of the above argument?
  • 0
    0

    Explanation

    The correct answer is D.

    The passage provides a rather one sided argument against giving to charity, exploring the options as to why this is negative on the market economy. It uses the words immoral to describe the charitable system, and also how giving to charity cannot be morally justified. Both of these extracts push you towards to correct answer of D being the best expression of the main conclusion, that giving to charity is immoral. Remember, that although you may not think giving to charity is immoral, this is the conclusion of the argument and therefore the correct answer.

    Post Comment

    Like other booming cities, including New York, London has thrived amid globalisation. It is growing by 100 000 new residents a year. However, if London has given striving people plenty of reasons to come, it has done a poor job of housing them. The median price of a London home has tripled over the past two decades. Since 2001 the disposable income of people renting property has fallen by 30 per cent as a result. Businesses are worried too. Commercial property in central London is twice as expensive as in New York. Productivity has begun to slow as promising firms are forced out. Overpriced property is costing London the economic and human diversity on which its prosperity depends.

    1. Which one of the following best expresses the main conclusion of the above argument?
  • 0
    0

    Explanation

    The correct answer is E.

    The article makes several points about property prices, yet the main overarching conclusion is the final line of the article where its details the effects of the increased property prices in causing the loss of economic and human diversity, of which prosperity depends. Hence the answer is E.

    Post Comment

    Young people are becoming ever more responsible. They are less likely to take drugs or drink than past generations. Very few have serious problems with alcohol or drugs. It is people in their forties who are most likely to be admitted to hospital for drinking-related problems. The latest figures on drug addiction show that 39% of heroin addicts are now over the age of forty, up from 19% in 2006. Fewer than ever are under the age of twenty-four. The number of young adults in prison is falling, but our overcrowded prisons are increasingly home to the older generation: between 2002 and 2013, the number of middle-aged men locked up increased by 130%. The same is true of suicide rates: on both sides of the Atlantic fewer young people but more middle-aged men are killing themselves.

    4. Which one of the following can be drawn as a conclusion from the above passage?
  • 1
    0

    Explanation

    The correct answer is B.

    All facts and figures in the article state that the young are more responsible and what can be considered as social problems such as drinking and drug taking is seen more in those over the age of forty (middle-aged). This in addition to the prison population and suicide rates being higher in middle-aged people all point to answer B being the correct conclusion that can be drawn from this paragraph as the overarching theme.

    Post Comment

    Schools have a duty to support the well-being of their students, and sport is one way to enhance this. Although many young people don’t enjoy the competitive side of sport, there are many physical activities that don’t require a competitive element, such as hillwalking and caving. Schools should offer these, or other non-competitive physical activities, as an alternative to traditional games lessons. Many people don’t continue to play sport after they leave school, yet physical fitness is important in maintaining general health and the ability to work. Introducing alternative physical activities that are attractive for less competitive adults is therefore an important part of preparing students for life after school.

    11. Which one of the following best expresses the main conclusion of the above argument?
  • 0
    0

    Explanation

    The correct answer is D.

    Points on many young not enjoying competitive sport, and the need to incourage people to continue playing sport after school all point to the main conclusion being that non-competitive alternatives to traditional games lesson should be offered by schools, answer option D.

    Post Comment

    Recently a pioneering natural flood management scheme slowed river peak flow by 15 to 20 per cent, saving a small town’s museum and several homes. As part of this scheme, special dams, designed to leak water gradually, were installed in nearby hills to slow the flow of water
    off the slopes. An upstream flood storage reservoir was installed, along with the planting of thousands of trees and the restoration of heather moorland, which acted as a sponge. Around half of the reduction in flood water in the town was due to the upstream land management measures, and half due to the effect of the flood storage area. Natural flood management measures improve water quality, prevent erosion and, in some cases, store carbon.

    15. Which one of the following can be drawn as a conclusion from the above passage?
  • 0
    0

    Explanation

    The correct answer is B.

    This article is talking about more than just reducing the risk for flooding. This is suggesting that the natural measures for flood managmenet are doing more than just reducing the flood risk. This leads you to the correct conclusion being drawn of answer option B.

    Post Comment

    People are quick to connect attempts to reduce the effects of climate change with the protection of the environment. However, responding to climate change is much more important for humanity than for nature. In the history of the planet, the relative stability of temperatures in the last ten thousand years is the exception rather than the norm. Nature has always found a way to adapt to large temperature variations but human civilisations, all of which have evolved in the last ten thousand years, have developed in a period with unusually reliable and predictable weather. Nature will find a way to move on, but climate change is potentially devastating for humans.

    27. Which one of the following best expresses the main conclusion of the above argument?
  • 0
    1

    Explanation

    The correct answer is B.

    The article states that nature has always found a way to adapt but human civilisations are less able to adapt to the changes as we have evolved during a stable climate with predictable weather. This is further supported by the final line of the article on how climate change could be devastating for humnas, but not nature. Therefore leading to the main conclusion being best expressed by answer option B.

    Post Comment

    Chiropractors rely heavily on manipulating their patients’ spines. This has benefits for some patients, but more than half of all patients suffer mild to moderate adverse effects after seeing a chiropractor. These are mostly local and referred pains that usually last for two to three days.
    Chiropractors often claim that these are necessary steps on the road to getting better. However, several hundred cases have been documented in which patients were seriously and often permanently damaged after chiropractic manipulations. What usually happens in these tragic instances is that, upon manipulation of the upper spine, an artery supplying the brain is overstretched and simply breaks up, leading to a stroke that can prove fatal.

    28. Which one of the following is a conclusion that can be drawn from the above passage?
  • 0
    0

    Explanation

    The correct answer is B.

    The tone of the article, although offering a balance of arguement, is heavily shifted to Chriopractors doing harm. This would narrow your options down to either A or B. As the article does not go as far to say Chiropractic therapy should be banned, or offer such strong opinions on the matter, you can conclude that the articles conclusions would be B, Chriopractic therapy may do more harm than good.

    Post Comment

    Nearly everyone thinks it is a bad thing to be a hypocrite, because it is profoundly dishonest to pretend to have moral standards that you do not actually have. But pretending you are better than you are is evidence that you still accept the moral principles you have failed to follow. So it is better than being shameless about your moral failings. Perceived in this way, hypocrisy can be considered to be a virtue. Being a hypocrite shows you feel a need to hide some aspect of your life because it would be too embarrassing to allow it into the open. Many hypocrites wish they were the people they pretend to be: so perhaps one day they will succeed.

    35. Which one of the following best expresses the main conclusion of the above argument?
  • 0
    0

    Explanation

    The correct answer is D.

    Although the article starts by talking about how hypocrisy is viewed by many as a bad thing, the article does go on to say how it can be considered a virtue, and explores this in detail. Therefore the only answer that follows this conclusion would be option D.

    Post Comment

    One of the greatest achievements of English grammar schools was the development and dissemination of a sophisticated system of standard punctuation for the English language. The complex rules for the use of commas conveyed a range of precise meanings, while the choice between semi-colon, colon and full stop expressed the exact, and often subtle, relationship between statements. Without this kind of attention to punctuation, there is bound to be a loss of precision in communication and in thinking. The current use of commas to join statements together, without any regard for grammar or the nature of the link between them, lacks rigour and covers up lazy thinking.

    39. Which one of the following can be drawn as a conclusion from the above?
  • 0
    0

    Explanation

    The correct answer is C.

    The article alludes to how correct teaching of punctuation benefited students. It does state that this was done through using grammar schools originally, however, it is important to note that the article is not suggesting that reintroducing grammar schools would solve this issue. Instead it talks of how we must pay more attention to grammer, leading us to the conclusion that answer option C must be correct over any other conclusion options (that don’t follow the main text).

    Post Comment

    An accident at the Fukushima nuclear power plant in Japan in 2011 caused the release of radioactive material, resulting in around half a million people being evacuated from the area. These people have not been allowed to return to the area because of the health risks of exposure to radiation. But the psychological consequences of long-term evacuation may be more significant than the physical effects of radiation. People who wish to return to live in the area around Fukushima should be allowed to. Studies of a similar situation at Chernobyl showed that average life expectancy for evacuees reduced – not predominantly because of cancer caused by radiation, but because of depression, alcoholism and suicide due to the impact of permanent evacuation. There is, moreover, massive disagreement in the scientific community over how much radiation is ‘safe’.

    45. Which one of the following best expresses the main conclusion of the above argument?
  • 0
    1

    Explanation

    The correct answer is D.

    This article, although starting with we evacuate nuclear power accident sites, focuses mostly on the negative effects seen by forcing evacuations. The article details to issues faced by individuals and how that the mental effects of evacuation has caused more negative effects than the radiation itself. Therefore, this article is aluding to how we should let individuals devide where they chose to live, to negate the mental health effects of evaucation, answer option D.

    Post Comment

    Official TSA Conclusions Questions (2008-2019) Review Screen

    Instructions

    Below is a summary of your answers. You can review your questions in three (3) different ways.

    The buttons in the lower right-hand corner correspond to these choices:

    1. Review all of your questions and answers.
    2. Review questions that are incomplete.
    3. Review questions that are flagged for review. (Click the 'flag' icon to change the flag for review status.)

    You may also click on a question number to link directly to its location in the exam.

    Official TSA Conclusions Questions (2008-2019) Section

    Final Answer Review Screen

    Instructions

    This review section allows you to view the answers you made and see whether they were correct or not. Each question accessed from this screen has an 'Explain Answer' button in the top left hand side. By clicking on this you will obtain an explanation as to the correct answer.

    At the bottom of this screen you can choose to 'Review All' answers, 'Review Incorrect' answers or 'Review Flagged' answers. Alternatively you can go to specific questions by opening up any of the sub-tests below.

    Official TSA Conclusions Questions (2008-2019) Section

    x

    +/-
    %
    MRC
    M-
    M+
    7
    8
    9
    4
    5
    6
    1
    2
    3
    ON/C
    0
    .
    ÷
    ×
    -
    +
    =

    Let's get acquainted ?
    What is your name?

    Next

    Nice to meet you, {{name}}!
    What is your preferred phone number?

    What is your preferred phone number?

    Next

    Just to check, what are you interested in?

    1-1 Tutoring Online Course Bursaries/Resources Other

    When should we call you?

    Another Day
    Skip the Call

    What time works best for you? (UK Time)

    8am-2pm 2pm-10pm
    10:00-10:30 10:30-11:00 11:00-11:30 11:30-12:00 12:00-12:30 12:30-13:00
    15:00-15:30 15:30-16:00 16:00-16:30 16:30-17:00 17:00-17:30 17:30-18:00 18:00-18:30 18:30-19:00 19:00-19:30 19:30-20:00

    How many hours of 1-1 tutoring are you looking for?

    0-5 10 20-30 40+

    My WhatsApp number is...

    Same as the one I entered Different to the one I entered

    For our safeguarding policy, please confirm...

    For our safeguarding policy, please confirm...

    I am under 18 I am over 18

    Which online course are you interested in?

    Next

    What is your query?

    Submit

    Sure, what is your query?

    Submit

    Loading...

    Thank you for your response.
    We will aim to get back to you within 12-24 hours.

    Lock in a 2 Hour 1-1 Tutoring Lesson Now

    If you're ready and keen to get started click the button below to book your first 2 hour 1-1 tutoring lesson with us. Connect with a tutor from a university of your choice in minutes. (Use FAST5 to get 5% Off!)

    Buy Now for £70